Sunteți pe pagina 1din 87

Advanced Taxation

Refresher Course
Success in Examinations
1. Planning
y Course Material
y Select the book which adequately covers the topic you are studying.
You may also select different books for different topics
y The key to selecting the right text book is consulting your teachers /
senior students. You may also refer the list of recommended reading
available on the website
y The All Essential Plan
y Panic makes you think less clearly, so avoid it by starting work early.
y Lecturers/tutors assume that you will decide for yourself what and
when to revise and may give little direction.
y A Good Plan Helps You:
y Identify if you are spending too much time on a topic
y Know what you have already done.
y Know what still needs to be done
y Prioritize things for effective studying.
ICMAP-STAGE 4, BUSINESS TAXATION REFRESHER From the desk of SOHAIL MERCHANT
`
F l t d th h hi f A ti d B i




Page 1
1. Planning
yFactors be Considered When Planning
y Study Sessions should be from one to three hours
y Have a definite break every hour
y Avoid late hours
y Revision for other papers at the same time
y Family commitments, relationships, friendships
y Contingencies such as illness
y How much sleep you need
y Plan recreation and relaxation into your time table
yMonitoring Your Plan
y Check your plan regularly to see how well you are doing. You
may need to amend your plan, e.g. if something unexpected
happens or if some revision takes longer than expected.
2. Preparation
Where to Study
Always in the same place
Choose a warm, light, well ventilated room
Away from other distractions
Properly furnished
Summarizing Key Points
u
and retain
Your notes should ideally be in the form of pointers which are easier to remember and
quicker to revise
Underline important points
Even if a paper involves mathematical calculation it is still very important that you study
the theory also to learn the concepts and logic behind the mathematical workings and
formulae.
Principles of Understanding
Always aim for understanding
Look for examples to illustrate the topic
Promote understanding by rearranging material, questioning the ideas and looking for
links with old ideas
Consider your topic from all possible angles
ICMAP-STAGE 4, BUSINESS TAXATION REFRESHER

From the desk of SOHAIL MERCHANT

`

For complete and thorough coaching of Accounting and Business




Page 2
2. Preparation
y Principles of Memorizing
y n
y Always try to link new material with what you have previously learnt
y Select the important items to remember
y Organize the material into a meaningful system
y The sequence of memorizing should be the same as the logical sequence of the
material
y Long pieces should be memorized in shorter chunks
y Go over notes, reading etc. within 12 hours of writing, reading etc.
y Try to master each topic before leaving it but do not spend so much time that
other areas or subjects are ignored
y Over learn. u stop when you have only just learnt something
y Start each session with a review of the previous session
y Mock Examinations
y At least 10-15 days before the end of the leave conduct real time mock
examinations
y Self assessment
y Identify weak areas
y Work on weak areas
y Go through the examiner comments
y Actually attempt the questions and do not just go through the solutions
3. Attempting the Paper
y Examination Techniques
y Controlling the anxiety is the key
y Arrive early at the exam to avoid panic. Be on your seat at least 10 minutes
before the examinations. This will reduce your anxiety and allow you to sort
out issues which may consume your time during the examinations.
y In the exam, spend the first 5 minutes glancing through the paper to make
sure you understand the instructions and to decide which questions to
answer first.
y Read the question very carefully until you know exactly what is required
y Note any special requirements e.g. list, detail, advise, explain, report, etc.
y Budget your time for each question in proportion to the marks given. Stop
working on it when that time is up, return to it if you have time to spare.
y Spending too much time on favorite topic at the expense of others may cost
you the exam
y Repetition of the same point using different descriptions does not fool the
examiner & only wastes time
y The first 50% of the marks of a particular question are the easiest to get; the
next 25% are harder; the last 25% are the hardest. If you run out of time: two
half answers may get more marks than one full one; jot down the main points
to include while they are in your mind and return later.
y Write clearly so the examiner can read your work. Number answers correctly.
ICMAP-STAGE 4, BUSINESS TAXATION REFRESHER

From the desk of SOHAIL MERCHANT

`

For complete and thorough coaching of Accounting and Business




Page 3
3. Attempting the Paper
y Y,d
y Possibilities for organizing your information in an examinclude:
y First plan your answer as to how you want to go ahead with your answer
y Give a clear opening paragraph, present information in a clear order, a final
paragraph drawing conclusions/summarizing. The opening paragraph should
be linked with final conclusions through one of the following ways:
y - step by step points where there is a sequence or stage
y - a main initial point to make an impact which you then develop
y - Putting different sides of an argument
y - Grouping theories/concepts through a theme
y Present your work well. Headings and a good layout make your work easier
to read
y Tables and graphs need to be clear with correct labeling
y Use practical examples to illustrate the points made subject to the
availability of time and requirements of the question. It may not be practical
to give examples where only brief answers are required
y As far as possible give answers in pointers showing the main heading and
then describing it in appropriate details as per the requirements of the
question. Just by giving pointers you can at least secure some marks and
convey your knowledge to the examiner.
3. Attempting the Paper
y ^dY,d
y It has been noted that most students only give the conclusions
in such type of questions
y The most important aspect of giving such questions is to test if
you have understood the concepts
y Therefore the key to such questions is the reasoning and not
the conclusion
y The examiner is interested in the thought process that went
into the conclusion.
y You can conclude correctly without any reasoning, by sheer
guessing you have a fifty percent chance of getting it right.
The examiner knows this and therefore no marks are allowed
for guessing the conclusion you must support it.
y If you have proper reasoning that forms the basis for your
conclusions you can at least get pass marks even if your
conclusion does not match with that of the examiner.
ICMAP-STAGE 4, BUSINESS TAXATION REFRESHER

From the desk of SOHAIL MERCHANT

`

For complete and thorough coaching of Accounting and Business




Page 4
3. Attempting the Paper
y How to Improve the Presentation of Your Scripts
y Marks that you will obtain for your answers depends on two factors:
y What you answered
y How you answered
y Start each new answer on a new page
y The arrangement should be pleasing to the eyes
y Write a fairly large and legible handwriting. But you should not try to change your style
just for the examinations. You will have to practice it before the examinations
y Write your headings boldly
y Use a dark ink and mediumpointed nib
y Leave space between subsections of answers
y The subject matter should be broken up into small paragraphs
y Use Apt sub-headings as it attracts the attention to the main divisions of the chapter
y The sentences should be short and crisp
y Make cancellations and corrections neatly
y Insert new words or sentences legibly and in an orderly way
y Watch your spelling and punctuation as it helps quick reading & prevent
misunderstanding
3. Attempting the Paper
y Most Commonly Made Mistakes
y Not resting adequately before the paper
y General instructions given on the answer scripts and sent
with the admit card are often ignored
y Questions are not read carefully
y Not planning before attempting the question
y Getting stuck over a single question
y Not clearly stating the assumptions used
y Not being quick enough
y Presentation and workings not clearly shown
y Students do not complete the paper more due to selective
studies and not because of the length of the paper
y Students tend to repeat points
y Irrelevant points are given
ICMAP-STAGE 4, BUSINESS TAXATION REFRESHER

From the desk of SOHAIL MERCHANT

`

For complete and thorough coaching of Accounting and Business




Page 5
3. Attempting the Paper
y
Coping with Nerves
y Stress can be good - it can make you mentally alert. You will do better
if you see stress as positive and the exams as a chance to show what
you can do, not as a way of tripping you up.
y Work out what to do if you panic. Take deep breaths
y Do good revision/preparation.
y Find out in advance as much as possible about the examination centre
or the examroom.
y Identify what to do in the first 5 minutes of the exam in what order
and stick to it.
y Make yourself comfortable for the exam (e.g., warm/cool clothes,
handkerchiefs, etc)
y Calm yourself beforehand (e.g., visualize a pleasant scene, distract
yourself)
y Avoid being overtired (is it worth staying up late to cramin extras?).
y Avoid last minute revision. Trying to remember facts then may block
out 'deep learning' (i.e. of concepts and principles).
Income Tax
yIncome
yAny bonus or bonus shares declared, issued or paid
by a company with a view to increase its paid-up
share capital shall not be an income in the hands of
the shareholder.
yDeductible Allowance
yWWF
yWPPF
yZakat paid under the Zakat and Ushr Ordinance,
1980.
y Zakat privately paid shall not be treated as deductible
allowance.
ICMAP-STAGE 4, BUSINESS TAXATION REFRESHER

From the desk of SOHAIL MERCHANT

`

For complete and thorough coaching of Accounting and Business




Page 6
Income Tax
yEmployment
yL includes:
yA directorship or any other office involved in the
management of a company;
yA position which entitles its holder to a fixed or
ascertainable remuneration; or
yThe holding or acting in any public office.
yGratuity & commutation of pension is exempt to an
employee to the extent provided in clause (13) of
Part-I of Second Schedule, but this exemption shall
not be available to any payment received from a
company by a director of such company who is not a
regular employee of such company.
Income Tax
yKibor
ykl8C8 means Karachi Inter-Bank Offered Rate prevalent
on the first day of each quarter of the financial year.
yMinor Child
yM C
tax year is under the age of eighteen (18) years. [2(33)]
yFor the purpose of section 90 (i.e., transfer of property)
M C
ICMAP-STAGE 4, BUSINESS TAXATION REFRESHER

From the desk of SOHAIL MERCHANT

`

For complete and thorough coaching of Accounting and Business




Page 7
Income Tax Rates
yIndividuals:
y Different tax rates for salaried and non-salaried taxpayers
y Marginal Tax Relief for salaried taxpayers only
yAssociation of Persons:
y 25% of taxable income
y Where turnover is 50M or above provisions of section 113
(i.e., minimum tax @ 1% of turnover ) shall also apply
yCompanies:
y Small companies 25% of taxable income
y Any other companies 35% of taxable income
y Provisions of section 113 (i.e., minimum tax @ 0.05% of
turnover ) shall also apply
Schemes of Taxation
yNormal Tax Regime (NTR)
y Income and tax liability are computed under normal
procedure by allowing admissible deductions, deductible
allowance, adjustment of losses, tax credits and tax rebates,
etc
ySeparate Taxation
y Certain incomes and transactions are not included in total
and taxable income; rather, are kept separate and charged to
tax at special rates
yFinal Tax Regime (FTR)
y Certain transactions are presumed as income and tax
deducted/collected at source is treated as full and final
discharge of tax liability in respect of income from such
transactions.
ICMAP-STAGE 4, BUSINESS TAXATION REFRESHER

From the desk of SOHAIL MERCHANT

`

For complete and thorough coaching of Accounting and Business




Page 8
Overriding Provisions
Section 3
The provisions of the Income Tax Ordinance shall apply
notwithstanding anything provided in any other law for
the time being in force.
Section 107
Where there is a contradiction between the provisions
l 1 C 1 1
provisions of the tax treaty shall apply.
Income from Assets
yGeneral Rule
y Income arising from any asset is the income of the person
who is owner of the asset
yException to the General Rule
y Where a persons transfers his asset under a revocable
transfer, or to his spouse (other than against consideration or
an agreement to live apart), a minor child, or to some other
person for their benefits, the income arising from such asset
shall be treated as income of the and not of the

yIncome fromSale of Land


y Taxable only if the person is in the business of buying and
selling of land
y Under all other cases not taxable under the Income Tax Law
ICMAP-STAGE 4, BUSINESS TAXATION REFRESHER

From the desk of SOHAIL MERCHANT

`

For complete and thorough coaching of Accounting and Business




Page 9
Taxation of Retailers (Ind. & AOPs)
y Retailers With Turnover Upto Rs. 5,000,000 [113A]
y A retailer whose total turnover for any tax year does not exceed Rs.
5,000,000 has two alternatives for determining his tax liability under the
Income Tax Ordinance. The alternatives available to him are:
y He may opt to pay tax computed on of his turnover during the tax year;
and
y He may opt to pay income tax under the normal tax regime (NTR).
Under this case, the taxable income and the tax liability shall be
computed as per normal procedure.
y Retailers with Turnover Exceeding Rs. 5,000,000 [113B]
y A retailer falling under this category is required to pay income tax on his
turnover instead of taxable income. Tax liability of such taxpayer shall be:
y Turnover upto Rs. 10 M - @ 0.5% of turnover
y Turnover exceeding Rs. 10 M Rs. 50,000 Plus 0.75% of exceeding
amount
y AOP with turnover of Rs. 50 M or above Minimum tax @ 1% u/s 113
y Company as a Retailer taxable under NTR
Permanent Establishment of Non-Resident
y The PE shall be treated as a distinct and separate entity from the non-
resident of which it is a PE. Its profit shall be computed on the basis of
this principle.
y Deduction on account of expenses (including executive and
administrative expenses) shall be allowed as per normal procedure.
y A PE shall not be allowed a deduction for any amount paid or payable
by it to its head office or to another PE on account of the following
expenses:
y Royalties, fees or other similar payments for the use of any
tangible or intangible asset.
y Compensation for any services (including management services).
y Profit on debt on money lent to the PE, except in connection with a
banking business.
y While determining the income, any amount which is received or
receivable by a PE from its head office or from another PE on account
of the following incomes shall not be taken into account:
ICMAP-STAGE 4, BUSINESS TAXATION REFRESHER

From the desk of SOHAIL MERCHANT

`

For complete and thorough coaching of Accounting and Business




Page 10
Permanent Establishment of Non-Resident
y Royalties, fees or other similar payments for the use of any
tangible or intangible asset.
y Compensation for any services (including management services).
y Profit on debt on money lent by the PE except in connection with a
banking business.
y Head office expenses shall be allowed as deduction equal to an
amount which is computed as below:
y Total head office expenses of non-resident Turnover of PE
y Total world wide turnover of non-resident
y Any excess amount allocated to PE shall not be allowed as deduction.
y A PE shall not be allowed a deduction on account of the following
expenses paid or payable by the non-resident;
y Any profit on debt to finance the operations of PE; or
y Any insurance premium in respect of the above stated debt.
Thin Capitalization
yThin capitalization is a situation wherein a company has
a very lesser amount of capital as compared to its
debts. A foreign-controlled resident company shall not
be allowed a deduction for the profit on debt paid by it
on that part of the debt as exceeds the prescribed ratio.
The provisions in this regard are as below:
y The company has a foreign debt to foreign equity ratio in
excess of three to one at any time during the tax year.
y Profit on debt shall be allowed as deduction if the debt-equity
ratio remains up to three to one. As and when it exceeds this
ratio, any amount paid as profit on that part of the debt as
exceeds three to one ratio shall not be allowed.
y The above provisions do not apply to:
y (i) A financial institution;
y (ii) A banking company; or
y (iii) A branch of a foreign company operating in Pakistan.
ICMAP-STAGE 4, BUSINESS TAXATION REFRESHER

From the desk of SOHAIL MERCHANT

`

For complete and thorough coaching of Accounting and Business




Page 11
For complete and thorough coaching of Accounting and Business
Thin Capitalization
y Foreign-Controlled Resident Company [106(2)]
y It means a resident company in which fifty percent (50%) or more
underlying ownership of the company is held by a non-resident person
either alone or together with an associate or associates.
y Foreign Debt [106(2)]
y Foreign debt has been defined in relation to foreign-controlled
resident company and it means the greatest amount of the total of
the following amounts:
y The balance of any debt owed by the company to a foreign controller or
his non-resident associates, the profit on which is either exempt from tax
or is taxable at a rate which is lower than the corporate rate; and
y The balance of any debt owed by the company to a person other than
specified in No. 1 above, if that other person is owing a similar amount of
debt to the foreign controller or his associates.
y Note: The greatest amount at any time in a tax year shall be
taken as foreign-debt.
y Foreign Equity [106(2)]
y The amount of foreign equity of a foreign-controlled resident
company shall be computed as below:
Thin Capitalization
y Amounts representing share in the equity of the company of
foreign-controller or his non-resident foreign associates:
y Paid up value of shares held XXX
y Share premium XXX
y Accumulated profit XXX
y Asset revaluation reserve XXX
XXX
y Less:
y Debt obligation owed to the company by the foreign-controller and
his non-resident associates XXX
y Share in accumulated losses XXX
XXX
y Foreign equity XXX
y Note: All the above mentioned amounts should be taken
at the balance appearing in the books at the beginning of the
tax year.






ICMAP-STAGE 4, BUSINESS TAXATION REFRESHER From the desk of SOHAIL MERCHANT
`
For complete and thorough coaching of Accounting and Business
Page 12
Modarabas
y Income from Non-Trading Activities
y According to clause (100) of Part-I of the Second Schedule the
income of a Modaraba from non-trading activities shall be
exempt from tax for any assessment year commencing on or
after 01-07-1999. In order to avail this exemption, it shall have
to fulfill the following conditions:
y Minimum 90% of the total profit (after transfer to mandatory
reserve) is distributed among the certificate holders; and
y For the purpose of determining the distribution of 90% profits, the
profits distributed through bonus certificates or shares shall not be
taken into account.
y Incomes from Trading Activities [Clause (18) of Part-II of
Second Schedule]
y Currently, a Modaraba is taxable for such incomes, which are
generated through trading activities. It shall be taxable at the
rate of twenty five percent (25%) of its total trading income
excluding the followings:
Modarabas
yDividend incomes;
yIncomes to which section 153 applies (i.e., supply of
goods, rendering of services or execution of contracts).
yIncomes to which section 154 applies (i.e., exports).
yNon-application of Minimum Tax u/s 113 [Clause
(11A)(xiii) of Part-IV of Second Schedule]
yThe provisions of sections 113 regarding payment of
minimum tax are not applicable to a Modaraba
registered under the Modaraba Companies and
Modaraba (Flotation and Control) Ordinance, 1980.
ICMAP-STAGE 4, BUSINESS TAXATION REFRESHER

From the desk of SOHAIL MERCHANT

`

For complete and thorough coaching of Accounting and Business




Page 13
Banking Companies
y No depreciation allowance or deduction shall be admissible
on assets given on finance lease
y Provisions for advances and off balance sheet items shall be
allowed up to a maximum of 1% of total advances; and
y Provisions for advances and off-balance sheet items shall be
allowed at 5% of total advances for consumers and small and
medium enterprises (SMEs)
y Provisioning in excess of 1% would be allowed to be carried
over to succeeding years
y If provisioning is less than 1% of the advances, then actual
provisioning for the year shall be allowed
y 1 -
the Prudential Regulations issued by the State Bank of
Pakistan shall not be allowed as expense
Banking Companies
y Capital Gain/Loss
y gains on sale of shares of listed shall be taxed
at the rate of ten per cent
y Where the shares of listed companies are disposed of within one
year of the date of acquisition, the gain shall be taxed at the
normal rate
y Loss on sale of shares of listed companies, disposed of within one
year of the date of acquisition, shall be adjustable against business
income of the tax year. Where such loss is not fully set off against
business income during the tax year, it shall be carried forward to
the following tax year and set off against capital gain only. No loss
shall be carried forward for more than six years immediately
succeeding the tax year for which the loss was first computed.
y Dividend Income
y The income under the head shall be taxed at the rate of
ten per cent
y Income fromBusiness Taxable @35%
ICMAP-STAGE 4, BUSINESS TAXATION REFRESHER

From the desk of SOHAIL MERCHANT

`

For complete and thorough coaching of Accounting and Business




Page 14
Banking Companies
yAdvance tax
y The banking company shall be required to pay advance tax for
the year under section 147 in twelve equal installments
payable by 15th of every month. Other provisions of section
147 shall apply as such.
y Provisions of withholding tax under this Ordinance shall not
apply to a banking company as a recipient of the amount on
which tax is deductible.
yMinimum Tax
y The provisions of section 113 shall apply to banking
companies as they apply to any other resident company
yExemptions
y Exemptions and tax concessions under the Second Schedule
to this Ordinance shall not apply to income of a banking
company computed under Seventh Schedule
Salary Income
ySalary Which Income:
y Employee / employer relation must exist between the
recipient and the payer
yDefinition:
y Salary, briefly, is any benefit to a person from his employer
yBasis of Taxing Salary:
y Primarily salary is taxable on actual receipt basis
y Exceptions are:
y Salary received in arrears may be taxed on accrual basis at the option
of taxpayer
y Commissioner may opt to tax salary income of an employee of a
private company if he is opinion that payment of salary was deferred
to avoid tax
y Terminal benefits may, at the option of taxpayer, be taxed at the ART
based on immediately preceding three tax years
ICMAP-STAGE 4, BUSINESS TAXATION REFRESHER

From the desk of SOHAIL MERCHANT

`

For complete and thorough coaching of Accounting and Business




Page 15
Salary Income
yLoan from Employer:
yInterest-free: Interest at BMR shall be salary income
yInterest-bearing: BMR less interest charged shall be
salary income
yInterest exceeding BMR: No benefit to employee
yBMR for tax year 2013 is 10% per annum
y Loan Used in Acquiring Asset Generating Taxable Income:
yInterest-free or interest up to BMR: Interest at BMR shall
be allowed as deduction against such income
yInterest exceeding BMR: Actual interest charged to the
employee shall be allowed as deduction against such
income
Salary Income
y Conveyance :
y Conveyance allowance Totally taxable
y Conveyance facility:
y For official use only Nothing is salary
y For personal use only 10% of cost is salary income
y For official and personal use 5% of cost is salary income
y If vehicle is on lease Instead of the lMv of the vehicle
at the time of its acquisition shall be used.
y House Accommodation:
y House rent allowance Totally taxable
y Rent-free accommodation:
y In big cities Higher of FMR or 45% of MTS/basic salary shall
be salary income
y In small towns Higher of FMR or 30% of MTS/basic salary
shall be salary income
ICMAP-STAGE 4, BUSINESS TAXATION REFRESHER

From the desk of SOHAIL MERCHANT

`

For complete and thorough coaching of Accounting and Business




Page 16
Salary Income
Medical Facility:
Medical allowance with no other medical facility -
Exempt up to 10% of basic salary. Any excess amount
shall be taxable
Hospitalization or Reimbursement of Medical Expenses:
Provided as per terms of employment Exempt
Provided without entitlement as per terms of employment
Fully taxable
Medical allowance as well as Free medical facility:
Medical allowance shall be taxable and facility shall be treated
as per general rule
Salary Income
Employee Share Option Scheme:
Value of right/option in itself is not taxable
If option exercised FMV of shares less price paid shall
be salary income
If option is renounced disposal consideration less price
paid for the option, if any, shall be salary income
FMV of the shares at the time of acquisition or when
free right to transfer those shares is granted shall be the
cost of acquisition of such shares. This shall be used for

ICMAP-STAGE 4, BUSINESS TAXATION REFRESHER

From the desk of SOHAIL MERCHANT

`

For complete and thorough coaching of Accounting and Business




Page 17
Income from Property
y Rent Chargeable to Tax (RCT)
y RCT shall include the following amounts:
y Higher of the rent received/receivable or the fair market rent for the
period for which the property was actually rented out;
y Forfeited deposit received under a contract for the sale of land or a
building;
y Any obligation of the owner paid by the tenant;
y One-tenth
y (1/10
th
) of the advance not adjustable against rent.
y Property Income is taxable on actual receipt basis or on
accrual basis
y It is a separate block of income chargeable to tax at special
rates applicable to it only
y Certain persons are required to deduct tax at the time of
making payment for Rent. This amount shall be adjustable
against final tax
Income from Business
yIncomes may fall under following categories:
y Local Supplies made out of:
y Own-Manufactured goods - NTR
y Locally purchased goods - FTR
y Imported goods - FTR
y Goods manufactured for exports and its scrap (maximum up to 20%
of such production) May be treated as Export (circular 20/92)
y Exports made out of:
y Own-Manufactured goods - FTR
y Locally purchased goods - FTR
y Imported goods Shall be excluded from exports, as tax u/s 148
shall be final tax in respect of such goods.
y Duty Draw-Backs
y Shall not be considered as additional receipt. The exports already
included it and was subject to tax deduction u/s 154. (circular 14/93)
ICMAP-STAGE 4, BUSINESS TAXATION REFRESHER

From the desk of SOHAIL MERCHANT

`

For complete and thorough coaching of Accounting and Business




Page 18
Income from Business
yDeductions Admissible
yAny expense incurred wholly and exclusively for the
business
yDepreciation and amortization
yExpense of death of animal being used for business other
than as stock-in-trade
yAmalgamation expenses by amalgamated company
yFinancial charges
yLease rentals by the lessee
yApportionment of Common deductions
yCommon expenditures shall be apportioned among
different heads of income on some reasonable basis
Income from Business
y Deductions Not Admissible [21]
y Any tax, cess or rate (including income tax) levied on the profits or gains
of the business.
y Any amount of tax deducted at source from an amount received by the
person.
y Any payment made to any person without deducting tax at source (under
section 149 to 158 & 232), if applicable. A person is required to deduct
tax at source in respect of the following payments:
y Salary;
y Rent;
y Brokerage or commission;
y Profit on debt;
y Payment to non-resident;
y Payment for services; or
y Fee.
y Any payment on which tax at source was deducted but has not been
paid.
y Any payment made by an association of persons to its partners or
members on account of profit on debt, brokerage, commission, salary or
any other remuneration.
ICMAP-STAGE 4, BUSINESS TAXATION REFRESHER

From the desk of SOHAIL MERCHANT

`

For complete and thorough coaching of Accounting and Business




Page 19
Income from Business
y Expenditure of a non- P C
L 1

y Total Head Office Expenditure Total Turnover in Pakistan
y Total World Turnover
y Any expenditure incurred on entertainment except those, which are
incurred:
y Abroad in connection with the business;
y In Pakistan on entertainment of foreign or local customers and suppliers;
y At the meetings of members, agents, directors and employees;
y On refreshment of employees;
y At the opening of branches; and
y On entertainment of persons related directly to business. [Rule-10]
y Notes:
y means the provision of meals, refreshments and reasonable
leisure facilities in accordance with the tradition of business and subject to
over all norms and customs of business in Pakistan.
y The Board may prescribe the conditions for allowing an entertainment
expense. The violation of such conditions shall also render the expense as
inadmissible.
Income from Business
y Any expenditure under a single account head exceeding Rs. 50,000 in aggregate
shall be inadmissible if the payment is not made through a crossed cheque or a
bank draft. However, this provision shall not be applicable to:
y Utility bills.
y Postage.
y Single transactions not exceeding Rs. 10,000.
y Payments on account of freight charges.
y A
reimbursement of expenses incurred on behalf of the taxpayer.
y Payments made to discharge any statutory obligation (such as duties, taxes,
octroi, export tax, fines, fee, cess, etc.).
y Note: Online transfer of payment from the business account of the
payer to the business account of the payee as well as payment through
credit card shall be treated as transactions through the banking channel, if
such transactions are verifiable from the bank statements of the respective
payer and the payee.
y Any payment on account of salary exceeding Rs. 15,000 per month if not made

ICMAP-STAGE 4, BUSINESS TAXATION REFRESHER

From the desk of SOHAIL MERCHANT

`

For complete and thorough coaching of Accounting and Business




Page 20
Income from Business
y Any contribution to such provident fund, pension fund,
gratuity fund, superannuation fund or annuity fund which is
not recognized or approved under the income tax law.
y Any contribution to any provident or other fund, if the person
has not made effective arrangements for deduction of tax at
source at the time of payments out of such fund.
y Any donation to an unapproved institution.
y Any provision against the profits of the business, e.g.,
provision for bad debts.
y Any appropriation of profit such as dividends, transfer to
reserves or capitalization in any way.
y Any expenditure in the nature of fine or penalty for the
violation of any law, rule or obligation.
y Any expenditure of a capital nature (e.g., purchase of assets).
y Any personal expenditure incurred by the person.
Income from Business
yBad Debts:
y Actual allowed; provision for doubtful debts, inadmissible
yLosses:
y Normal admissible
y Abnormal admissible (net-off any claim)
yNon-payment of a liability:
y Income if not paid within three years.
y Expense if paid thereafter
yC Loan by HBFC/NBFCs:
y Provision equal to 3% of income from such loan shall be
deduction on account of bad debts
y If actual bad debts are more, those shall be carried forwarded
to next year for adjustment of provision for that year
ICMAP-STAGE 4, BUSINESS TAXATION REFRESHER

From the desk of SOHAIL MERCHANT

`

For complete and thorough coaching of Accounting and Business




Page 21
Income from Business
yAssets:
y Depreciable asset Normal depreciation (DBM)
y Eligible depreciable asset Initial allowance plus normal
depreciation (DBM)
y Intangible Amortization for lesser of useful life of the asset
of 10 years (SLM)
y Pre-commencement expenditure Amortized @ 20% of cost
(SLM)
yAcquisition of an Asset:
y A person shall be treated as having acquired an asset at any of
the following times:
y When he begins to own an asset;
y When he is granted any right to own an asset; or
y When a personal asset is applied for business use.
Income from Business
y Disposal of An Asset
y Disposal of an asset means to pass over an asset to some other person.
In its general meanings it denotes the change in ownership of an asset.
Under the Income Tax Ordinance, 2001 a disposal may take place when
an asset is: [75(1)]
y Sold;
y Exchanged;
y Transferred;
y Distributed;
y Cancelled;
y Redeemed;
y Relinquished (to abandon, give up or renounce some right or thing);
y Destroyed;
y Lost;
y Expired;
y Surrendered;
y Transmitted by succession or under a will;
y In case of a business asset, applied to personal use; or [75(3)]
y In case of a business asset, discarded or ceased to be used in business.
[75(3A)]
ICMAP-STAGE 4, BUSINESS TAXATION REFRESHER

From the desk of SOHAIL MERCHANT

`

For complete and thorough coaching of Accounting and Business




Page 22
Income from Business
yAsset from Foreign Currency
y Where an asset is acquired with a loan in foreign currency
and exchange rate fluctuation increases or decreases the
liability of the person in Pak rupees, then any increase or
decrease in the liability, before full and final repayment of the
loan shall also be added to or deducted from the cost of the
asset.
yAsset Acquired FromAny Subsidy, Etc.
y The amount of any grant, subsidy, rebate, commission or any
other assistance received or receivable in respect of
acquisition of an asset shall not be included in the cost of
such asset. However, where the amount of grant, etc., is
chargeable to tax under the Income Tax Ordinance, 2001 then
such amount shall also be included in the cost of the asset.
Income from Business
y Consideration of a Leased Asset
y The residual value received by a leasing company on maturity of a lease
agreement shall be taken as consideration for disposal of such asset.
y However, it should be noted that the residual value plus total amount
received by the leasing company towards the cost of the asset (i.e. the
principal part of the lease rentals realized by to the leasing company) should
not be less than the original cost of the asset to the leasing company.
y Consideration of Assets Sold in Bulk
y Where different assets are disposed off through a single transaction and the
consideration of each asset is not determined / specified separately, the
total consideration received shall be apportioned amongst all assets so
disposed off. This apportionment shall be on the basis of fair market value of
assets at the time of the transaction. In other words the disposal
consideration shall be computed with the help of the following formula:
y Total Consideration Received FMV of an asset
y FMV of all assets
ICMAP-STAGE 4, BUSINESS TAXATION REFRESHER

From the desk of SOHAIL MERCHANT

`

For complete and thorough coaching of Accounting and Business




Page 23
Income from Business
y Non-recognition of Gain or Loss on Disposal of an Asset [79]
y Normally, the gain or loss on disposal of an asset is taken into account. However,
under the following cases no gain or loss shall be taken to arise, if the person
acquiring the asset is a resident person:
y Where disposal is between spouses under an agreement to live apart;
y Where disposal is by reason of the transmission of the asset to an executor or
beneficiary on the death of a person;
y Where disposal is by reason of a gift of the asset;
y Where disposal is by a company to its members on its liquidation;
y Where disposal is by an association of persons to its members on its
dissolution. However, under this case the assets should be distributed in
accordance with the interest of members in the capital of AOP.
y Notes:
y If the person acquiring the asset is a non-resident person, the gain or loss on
disposal of assets shall be computed as per normal procedure.
y Under all the above cases it shall be treated that:
y The person is acquiring the asset of the same character as the person disposing
of the asset; and
y The person is acquiring the asset at a cost that is equal to the cost of the asset
at the time of disposal to the person disposing it of.
y Where the disposal is by reason of the compulsory acquisition of the asset under
any law. In order to avail this benefit the consideration received on disposal shall be
reinvested (within one year of the disposal) in an asset of similar kind.
Income from Business
yExceptions Regarding Cost & Disposal Consideration
y Passenger transport vehicle not plying for hire:
y The cost of such vehicle shall be lesser of the actual cost of the
vehicle or Rs. 1,500,000. In other words the maximum cost for
depreciation purpose shall be Rs. 1,500,000 if the vehicle is
purchased at a price higher than this amount.
y For gain or loss on disposal of a passenger transport vehicle not
plying for hire the Received on shall be
computed according to the following formula
y A B
y C
y A = Amount received on disposal of vehicle
y B = Cost determined for depreciation purpose (i.e., lesser of actual
cost of the vehicle or Rs. 1,500,000)
y C = Actual cost of acquiring the vehicle.
y Assets given on lease by leasing company, etc:
ICMAP-STAGE 4, BUSINESS TAXATION REFRESHER

From the desk of SOHAIL MERCHANT

`

For complete and thorough coaching of Accounting and Business




Page 24
Income from Business
yDisposal of immovable property:
y Any consideration received on disposal of an immovable
property shall be treated as cost of the property if the
consideration exceeds the original cost of the asset. Under
such a case the total amount allowed as deduction on account
of depreciation allowance (accumulated depreciation) on such
asset shall become the gain on disposal of the asset.
yExport of depreciable asset:
y The cost of the asset shall be treated as the consideration
received on disposal of an asset if:
y It is a depreciable asset;
y It has been used in Pakistan by the person; and
y It is exported or transferred by the person out of Pakistan
Capital Gains
yCapital Asset [2(10) & 37(5)]
y Capital asset means property of any kind held by a person
excluding the following assets:
y Any stock-in-trade, consumable stores or raw materials;
y Any depreciable assets;
y Any intangible asset on which amortization is allowed u/s 24;
y Any immovable property; and
y Any movable property held for personal use by the person or his
family member dependent upon him. However, the following assets
shall be treated as capital assets:
y A painting, sculpture, drawing or other work of art;
y Jewelry;
y A rare manuscript, folio or book;
y A postage stamp or first day cover;
y A coin or medallion; or
y An antique.
ICMAP-STAGE 4, BUSINESS TAXATION REFRESHER

From the desk of SOHAIL MERCHANT

`

For complete and thorough coaching of Accounting and Business




Page 25
Capital Gains
yTax on Capital Gains [Clause (5B)]
yCapital gain is included in taxable income and taxed
under NTR
yWhere a capital asset is disposed of after one year of its
acquisition then the gain for income tax purposes shall be
taken as 3/4
th
(i.e., 75%) of the actual gain on disposal.
Remaining 1/4
th
(i.e., 25%) of the gain shall be treated as
exempt.
yDisposal within one year of acquisition total capital gain
in taxable
yTax @ 10% of the capital gain shall be charged if the gain
is from the sale of shares or assets by a private limited
company to Private Equity and Venture Capital Fund
Capital Gains
yCapital Gain on Disposal of Securities [37A]
y From 1st day of July, 2010 capital assets have been split into
two categories for taxation purposes. A new category termed
as has been introduced. ^ means the
following capital assets:
y Share of a public company;
y Voucher of Pakistan Telecommunication Corporation;
y Modarba certificate;
y An instrument of redeemable capital:
y Participation Term Certificates (PTCs);
y Term Finance Certificates (TFCs);
y Musharika Certificates; and
y Any other security (other than shares) not based on interest; and
y Derivative products (e.g., treasury bonds).
ICMAP-STAGE 4, BUSINESS TAXATION REFRESHER

From the desk of SOHAIL MERCHANT

`

For complete and thorough coaching of Accounting and Business




Page 26
For complete and thorough coaching of Accounting and Business
Capital Gains
y Taxation of Gain of Securities for Tax Year 2013
ycapital gain arising from disposal of held for a
period of less than a year shall be treated as a separate
block of income and charged to tax depending upon the
holding period of security
y Loss on Disposal of Security
yAny loss sustained by a person on disposal of
shall be treated separately from any other loss sustained by
him. This loss shall be dealt with as below:
yAny loss sustained on disposal of a in a tax
year shall be set-off only against any gain of the person
from any other disposed off during that tax
year; and
yLoss on disposal of a shall not be carried
forward to the subsequent tax year.
Income from Other Sources
Incomes Covered:
All incomes except the following shall be taxable
under this head:
Incomes which are covered under any other head
of income
Incomes covered under separate taxation
Incomes covered under FTR
Incomes exempt from tax
Deductions:
All expenses incurred for the purpose of earning this
income shall be allowed as deduction
ICMAP-STAGE 4, BUSINESS TAXATION REFRESHER

From the desk of SOHAIL MERCHANT

`





Page 27
Dividend
Received by a Company:
Taxable as a separate block of income @ 10% of
the gross amount of dividend
Received by Any Other Person
Taxable under FTR
Obligation of Company Paying Dividend:
Tax shall be deducted at the time of making
payment for the dividend u/s 150
Tax Credits
yForeign Tax Credit
y Salary income
y Any other income
yDonations to Approved Institutions/Funds
y Specified under clause (61) of Part-I of Second Schedule
y Others
y Maximum limit of 20% or 30% not applicable in case of
donations to Agha Khan Hospital and Medical College
yInvestment in Shares
y Lesser of cost, 10% of taxable income or Rs. 300,000
yContribution to Approved Pension Fund
y Lesser of contribution, 20% of taxable income or Rs. 500,000
yMark-up on Loans for Houses
y Lesser of profit of debt, 50% of taxable income or Rs. 750,000
ICMAP-STAGE 4, BUSINESS TAXATION REFRESHER

From the desk of SOHAIL MERCHANT

`

For complete and thorough coaching of Accounting and Business




Page 28
Tax Credits
yTax Credit To Manufacturer Registered Under Sales Tax
Act
y A manufacturer shall be entitled to a tax credit of two and a
half per cent (2 %) of tax payable for a tax year if the
following conditions are satisfied:
y The manufacturer is registered under the Sales Tax Act, 1990;
y Ninety per cent (90%) of the sales of the manufacturer are to the
persons who are registered under the Sales Tax, 1990;
y In order to claim the tax credit the person shall provide complete
details of the persons to whomsales were made;
y The person should be taxable under normal tax regime. Tax credit
shall not be allowed to a person whose income is covered under final
tax or minimumtax; and
y Carry forward of any amount shall not be allowed where full credit
may not be allowed against the tax liability for the tax year.
Tax Credits
y Tax Credit For Investment In Plant And Machinery [65B]
y A tax credit equal to 10% of the amount invested by a company in purchase of plant and
machinery shall be allowed against the tax payable by it. Other provisions in this regard
are:
y The plant and machinery is purchased by the company for the purposes of
Balancing, Modernization and Replacement (BMR) of the plant and machinery
already installed.
y The plant and machinery is for an industrial undertaking set up in Pakistan and is
owned by the company making the investment.
y The plant and machinery should be purchased between 01-07-2010 and
30-06-2015.
y Amount of tax credit shall be deducted from the tax payment in the tax year in
which plant or machinery is purchased and installed.
y Where there is no tax payable for the year in which plant or machinery is installed
or tax payable for that year is less than the amount of tax credit, the unadjusted tax
credit shall be carried forward and deducted against tax payable for following tax
years.
y The amount of unadjusted tax credit may be carried forward maximum up to two
(2) tax years.
y The Commissioner may re-compute the tax payable for the relevant tax years if
subsequently it is found by the CIR that any of the above-referred conditions was
not fulfilled. Under this case it shall be deemed that tax credit was wrongly allowed.
ICMAP-STAGE 4, BUSINESS TAXATION REFRESHER

From the desk of SOHAIL MERCHANT

`

For complete and thorough coaching of Accounting and Business




Page 29
Tax Credits
yTax Credit For Enlistment [65C]
y A tax credit @ 5% of tax payable shall be allowed to a
company for the tax year in which it is enlisted in any
registered stock exchange in Pakistan.
yPriority Of Tax Credits [4(3)]
y Where a person is entitled to more than one tax credit,
the tax credit shall be allowed in the following order:
y First of all foreign tax credit; then
y Tax credit for investment, donations, etc., allowable u/s 61-
64; and then
y Tax already deposited by or on behalf of the taxpayer.
Withholding Tax
yImports u/s 148
y Final discharge except under certain specified cases
ySalary u/s 149
y Adjustable against final tax liability
yDividend u/s 150
y Final discharge except in case of a company receiving it
yProfit on debt u/s 151
y Final discharge except received on Government securities,
where it is adjustable
yNon-Residents u/s 152
y FTR if deducted against Royalty, Fee for technical services,
construction, services or advertisement contracts, insurance
or re-insurance premium
y NTR in case of all other payments to non-residents
ICMAP-STAGE 4, BUSINESS TAXATION REFRESHER

From the desk of SOHAIL MERCHANT

`

For complete and thorough coaching of Accounting and Business




Page 30
Withholding Tax
Goods u/s 153
l S l18
On own manufactured goods NTR
Services u/s 153
For companies NTR
For AOPs Minimum tax on such income
For Individuals Minimum tax on such income
Contracts u/s 153
Covered under FTR
Withholding Tax
Exports u/s 154
FTR
Rent u/s 155
Adjustable
Prizes u/s 156
FTR
ICMAP-STAGE 4, BUSINESS TAXATION REFRESHER

From the desk of SOHAIL MERCHANT

`

For complete and thorough coaching of Accounting and Business




Page 31
Sales Tax
yCottage Industry
y C means a manufacturer who fulfills any of
the following conditions:
y Whose annual turnover from taxable supplies during last
twelve (12) months does not exceed Rs. 5,000,000; or
y Whose annual utility bills (i.e., electricity, gas and telephone)
during last twelve (12) months do not exceed Rs. 700,000.
yGoods
y C
following:
y Actionable claims;
y Money;
y Stocks;
y Shares; and
y Securities.
Sales Tax
y Input Tax
y In relation to a registered person, the means:
y The tax levied under Sales Tax Act, 1990 on the supply of goods to the person;
y The tax levied under Sales Tax Act, 1990 on import of goods by the person;
y In relation to goods or service acquired by the person, excise duty levied under
Federal Excise Act, 2005 in sale tax mode on the manufacture or production of
goods, or rendering or providing of services;
y The Provincial Sales Tax levied on services rendered or provided to the person;
and
y The tax levied under Sales Tax Act, 1990 of Pakistan as adapted in the State of
Azad Jammu and Kashmir on the supply of goods received by that person;
y Output Tax
y In relation to a registered person means the following taxes
and duties payable by that person:
y Tax levied under the Sales Tax Act, 1990 on supply of goods made by that
person;
y Excise duty levied under the Federal Excise Act, 2005 in sales tax mode on
manufacture or production of goods or the rendering or providing of the
services by the persons; and
y The Provincial Sales Tax levied on services rendered or provided by the person.
ICMAP-STAGE 4, BUSINESS TAXATION REFRESHER

From the desk of SOHAIL MERCHANT

`

For complete and thorough coaching of Accounting and Business




Page 32
Sales Tax
y Retail Price
y It means a price fixed by the manufacturer, inclusive of all duties, charges
and taxes (other than sales tax) at which any particular brand or variety
of any article should be sold to the general consumer.
y Where more than one such price is fixed for the same brand or variety,
the highest of such prices shall be taken as retail price.
y Supply
y S
owner. It also includes the following:
y Sale or transfer under a hire purchase agreement;
y Putting to private, business or non-business use of the goods
acquired, produced or manufactured in the course of taxable activity
for purposes other than those of making a taxable supply;
y Auction or disposal of goods to satisfy a debt owed by a person; and
y Possession of taxable goods held immediately before a person
ceases to be a registered person.
Sales Tax
yTime Of Supply
y1 of is defined separately for supply of goods
and rendering of services.
ySupply of Goods
y 1 of in relation to a supply of goods (other than
under hire purchase agreement) means the time at which the
goods are delivered or made available to the recipient of the
supply.
ySupply under Hire Purchase Agreement
y Where goods are supplied under hire purchase agreement,
time of supply means the time at which agreement is entered
into.
yRendering of Services
y 1 of in relation to services, means the time at
which the services are rendered or provided.
ICMAP-STAGE 4, BUSINESS TAXATION REFRESHER

From the desk of SOHAIL MERCHANT

`

For complete and thorough coaching of Accounting and Business




Page 33
Sales Tax
y Value Of Supply
y It means the consideration in money including all Federal and Provincial duties
and taxes which a supplier receives form the recipient against taxable supplies.
While determining the value of supply the amount of sales tax is not included.
y Value of Supply Where Trade Discount is Allowed
y Where a trade discount is allowed the value of supply will be the
discounted price (prior to the amount of sales tax). In this case the
following conditions shall be fulfilled:
y The discount allowed is in conformity with the normal business practice; &
y The tax invoice shows the discounted price and the related tax.
y Open Market Price as Value of Supply
y Open market price will be taken as value of the supply in the following cases:
y Where the consideration of supply is in kind or partly in kind and partly in
money;
y Where the supplies are made to an associated person against no
consideration or for a consideration, which is lower than the open market
price; and
y Where for any special nature of a transaction it is difficult to ascertain the
value of a supply.
Sales Tax
Value in Case of Supply on Installment Basis
Where a taxable supply is made to a consumer from general
public on installment basis and the price includes a mark-up or
surcharge, then the value of supply shall be the open market
price (excluding the amount of tax on such supply).
Value of Supply for Imported Goods
Where the goods are imported the value determined under
section 25 of the Customs Act, 1969 will be the value of the
supply. The amount of customs duties and Federal excise duty
levied thereon will also be included in the value of the supply.
ICMAP-STAGE 4, BUSINESS TAXATION REFRESHER

From the desk of SOHAIL MERCHANT

`

For complete and thorough coaching of Accounting and Business




Page 34
Sales Tax
yZero Rated Supply
yIt means a supply, which is charged to tax at the rate of
zero percent under section 4 of the Sales Tax Act, 1990.
yAccording to section 4, the following goods shall be
charged to tax at the rate of zero percent:
y Goods exported out of Pakistan;
y Goods specified in the Fifth Schedule;
y Supply of stores and provisions for consumption aboard a
conveyance proceeding to a destination outside Pakistan;
y Such other goods as the Federal Government may notify; and
y Such other goods as may be specified by the Board through a
general order as are supplied to registered persons engaged in
manufacture and supply of zero-rated goods.
Sales Tax
ySales Tax
y The Sales Tax is charged, levied and paid at the rate of
seventeen per cent (17%) of the value of taxable supplies
made by a registered person and of the value of goods
imported into Pakistan. The sales tax, being an indirect tax, is
ultimately shifted towards the ultimate consumers of the
goods. Tax in respect of imported goods is paid at the time of
clearance of goods from the Customs authorities. Tax for the
taxable goods supplied in Pakistan is paid at the time of filing
of return. Under this case tax for a tax period is computed as
under:
y Output tax XXX
y Less: Input tax (adjustable against output tax) XXX
y Tax payable / (Refundable) for the tax period XXX
ICMAP-STAGE 4, BUSINESS TAXATION REFRESHER

From the desk of SOHAIL MERCHANT

`

For complete and thorough coaching of Accounting and Business




Page 35
Sales Tax
y Tax On Supplies Specified In Third Schedule
y Taxable supplies specified in the Third Schedule shall be liable to tax @
17% of the retail price.
y Tax By Importers
y The v Addition 1 shall be levied and collected at import stage on
import of goods @2% of the value of goods in addition to tax chargeable
u/s 3 of the Sales Tax Act (i.e., 17%) or a notification issued for this
purpose.
y If the goods are imported by a manufacture for in-house consumption
then value addition tax shall not be charged. [R 58B(1)]
y Value addition tax paid on imports shall be treated as input tax and
adjustable against output tax and be carried forward as specified in the
Sales Tax Act.
[R-58B(2)]
y Any excess amount of input tax over output tax, which is attributable to
value addition tax, shall in no case be refunded to the registered person.
y Where the person is also dealing in goods other than imported goods, he
shall be entitled to file refund claim of excess carried forward input tax
for a period as per section 10 and the rules, after deducting the amount
attributable to the tax paid at import stage. Such deducted amount may
be carried forward to subsequent tax period. [R-58C]
Sales Tax
yConditions for Claiming Credit of Input Tax
yA registered person can deduct input tax from output tax
only if:
y He holds a tax invoice in respect of taxable supplies made in
Pakistan,
y He holds a goods declaration duly cleared by the Customs, in
case of imported goods, or
y He holds a treasury challan showing payment of sales tax, in
respect of goods purchased in auction. The challan shall bear
the name and registration number of the person. [7(2)]
y Note: All above referred documents (i.e., tax invoice,
goods declaration or treasury challan) should bear the name
and Sale Tax registration number of the person claiming the
credit of input tax.
ICMAP-STAGE 4, BUSINESS TAXATION REFRESHER

From the desk of SOHAIL MERCHANT

`

For complete and thorough coaching of Accounting and Business




Page 36
Sales Tax
y Restriction of Credit of Input Tax
y A registered person is not entitled to deduct or reclaim the input tax if it
is paid in respect of the following goods: [8(1)]
y The goods, which are not used for manufacture or production of
taxable goods or for making taxable supplies;
y The goods in respect of which extra tax [u/s 3(5)] is applicable. Only
extra tax will not be allowed as input tax;
y The goods in respect of which sales tax has not been deposited in
the government treasury by the respective supplier;
y The goods which are exempt from tax;
y Input tax paid on fake invoices;
y Purchases made by such registered person, who has not furnished
the details as required by Board through a notification issued under
section 26(5);
y Supply of specified goods to non-registered person; and
y Any other goods, which are specified for this purpose by the Federal
Government.
Sales Tax
y Adjustable Input Tax [8B]
y In relation to a tax period, a registered person shall not be allowed to
adjust or refund input tax in excess of ninety per cent (90%) of the output
tax for that tax period. In other words at least ten per cent (10%) of the
output tax for a tax period must be paid by a registered person.
y Tax charged on the acquisition of fixed assets shall be adjustable against
the output tax in twelve (12) equal monthly installments. [8B(1)]
y The Board may, by notification, exclude any person or class of persons
fromthe purview of the above discussed provisions. [8B(1)]
y The inadmissible input tax (i.e., over and above 90% of output tax) may
be allowed as adjustment or refund subject to the following conditions:
[8B(2)]
y In Case of Companies: Upon furnishing a statement along with annual audited
accounts, duly certified by the auditors, showing value additions less than 90%
of the output tax; or
y In Case of Other Registered Persons: If conditions and restrictions as may be
notified by the Board are satisfied.
y The adjustment or refund of input tax as discussed in point No. 4, if any,
shall be made on yearly basis in the second month following the end of
the financial year of the registered person. [8B(3)]
ICMAP-STAGE 4, BUSINESS TAXATION REFRESHER

From the desk of SOHAIL MERCHANT

`

For complete and thorough coaching of Accounting and Business




Page 37
Sales Tax
y Apportionment of Input Tax
yRules-24 and 25 of the Sales Tax Rules, 2006 deal with
the apportionment of input tax. The provisions of these
Rules are summarized below.
y These rules are applicable to such registered persons who
make taxable and exempt supplies simultaneously.
y Input tax paid on raw materials relating wholly to the taxable
supplies shall be admissible.
y Input tax paid on raw materials relating wholly to the exempt
supplies shall not be admissible.
y Input tax incurred for making both exempt and taxable
supplies shall be apportioned and admissible amount shall be
calculated as below:
y Residual Input Tax Value of Taxable Supplies
y Value of Taxable + Exempt Supplies
Sales Tax
y Debit Credit Notes
y A registered person may issue a debit or credit note and
adjust the resultant modification in the tax amount against
the output tax if the following conditions are fulfilled:
y The person has issued a tax invoice against the supply of goods
made by him; and
y The modification is necessary due to any of the following reasons:
y Cancellation of supply;
y Return of goods;
y Change in the nature of supply;
y Change in the value of supply; or
y Any other event requiring modification in the amount of tax on invoice and in
the return
ICMAP-STAGE 4, BUSINESS TAXATION REFRESHER

From the desk of SOHAIL MERCHANT

`

For complete and thorough coaching of Accounting and Business




Page 38
y
Assessment of Tax
y Under the following circumstances an authorized officer of
Inland Revenue shall make an order for assessment of tax,
including imposition of penalty and default surcharge:
y Where a person who is required to file a return has failed to file it by
the due date;
y Where a person, due to some miscalculations, pays a lesser amount
than actually payable;
y Where a person has not paid the tax due on supplies made by him;
and
y Where a person has claimed a credit or refund of an input tax which
was not admissible under the Sales Tax Act, 1990.
y The officer of Inland Revenue shall give a show-cause notice
(within five (5) years) and provide an opportunity of being
heard to a person before making an order for assessment of
tax, etc. Where a person files the return after due date and
pays the amount of tax payable along with the default
surcharge and penalty, the show-cause notice and the order
of assessment shall abate.
Sales Tax
ySale Of Taxable Activity, Etc.
y Transfer to Non-Registered Person [49(1)]
y If the taxable activity is terminated or transferred to a non-
registered person, the stock of taxable goods shall be
deemed as supply and the tax on such goods shall be
recovered fromthe registered person.
y Where the tax could not be recovered from the transferor, it
shall constitute a first charge on the assets of the business
and shall be recovered fromthe transferee.
y Transfer to Another Registered Person [49(2)]
y If the taxable activity is sold or transferred to another
registered person as an ongoing concern, the tax chargeable
on stocks of goods shall be paid by the transferee.
ICMAP-STAGE 4, BUSINESS TAXATION REFRESHER

From the desk of SOHAIL MERCHANT

`

For complete and thorough coaching of Accounting and Business




Page 39
Sales Tax
y Inadmissible Transactions [73]
y Payment of the amount of invoice of a transaction (other than
utility bills) as shown in the sales tax invoice should be made
through crossed cheque, bank draft, pay order or any other
banking instrument if it exceeds Rs. 50,000. Otherwise, such
transaction shall not be admissible for the purposes of input tax
credit, adjustment or deduction, or refund, repayment or draw
back or zero rating of tax under the Sales Tax Act.
y In order to render a transaction (exceeding Rs. 50,000) as
admissible the following conditions should be fulfilled:
y The payment of tax is made through a banking channel/instrument.
y The banking instrument should be in favour of the supplier.
y It should be drawn from the business bank account of the buyer to
the business bank account of the supplier.
Sales Tax
y Tax Paid on Stocks Acquired Before Registration [59]
y Where a person has purchased from a registered person or has
imported certain goods before his registration under the Sales Tax
Act, 1990, the sales tax paid by him on such purchases or imports
shall be deemed as input tax if the following conditions are fulfilled:
y In Case of a Purchase Froma Registered Person
y The goods were purchased within thirty (30) days before the date of
compulsory registration or the date of application for registration.
y The person holds a proper tax invoice issued by the seller.
y The goods are verifiable as unsold or un-consumed stocks on the date of
compulsory registration or the date of application for registration.
y In Case of an Import of Goods
y The goods were imported within ninety (90) days before the date of
compulsory registration or the date of application for registration.
y The person holds a of in respect of such import.
y The goods are verifiable as unsold or un-consumed stocks on the date of
compulsory registration or on the date of application
ICMAP-STAGE 4, BUSINESS TAXATION REFRESHER

From the desk of SOHAIL MERCHANT

`

For complete and thorough coaching of Accounting and Business




Page 40
Provincial Sales Tax Ordinances
y Scope of Tax
y The sales tax on services is payable @ 17% of the value of taxable
services rendered or provided in a province.
y Applicability of Provisions of Sales Tax Act, 1990
y The sales tax on services under the Provincial Sales Tax
Ordinances is levied and collected under the Sales Tax Act, 1990.
The provisions of the Act and Rules, etc., made under the Act
shall apply in respect of the following matters:
y Manner, time and mode of payment;
y Registration and de-registration;
y Keeping of records and audit;
y Enforcement and adjudication;
y Penalties and prosecution; and
y All other allied and ancillary matters.
Registration
yPersons Liable To Be Registered [Rule-4]
y The following persons are required to get themselves
registered under the Sales Tax Act, 1990:
y A manufacturer not being a cottage industry;
y A retailer whose value of supplies exceeds rupees five (5) million in
any period during the last twelve months;
y An importer;
y A wholesaler, dealer and distributor;
y A person who is required under any law (whether Federal or
Provincial) to be registered for the purpose of any duty or tax
collected or paid as if it were a levy of sales tax to be collected under
the Sales Tax Act; and
y Examples of such laws are Federal Excise Act, 2005 and Provincial Sales Tax
Ordinances.
y A commercial exporter who intends to obtain sales tax refund
against his zero-rated supplied.
ICMAP-STAGE 4, BUSINESS TAXATION REFRESHER

From the desk of SOHAIL MERCHANT

`

For complete and thorough coaching of Accounting and Business




Page 41
Registration
y De-Registration [21 & Rule-11]
y Under the following circumstances the CRO or other authorized officer
may, on an application made by a registered person to CRO, cancel the
registration fromsuch date as may be determined by him:
y If a registered person ceases to carry on his business.
y If the supplies made by a registered person become exempt fromthe sales tax.
y If the turnover of a registered person during the last twelve months remains
below the following limits.
y In case of a manufacturer or a producer, taxable turnover of five (5) million
rupees; and
y In case of a retailer, total turnover of five (5) million rupees.
y Where an exporter had got registration for the purpose of claiming refunds on
the basis of zero-rating export, he may like to de-register upon ceasing to be
an exporter.
y The CRO may of its own (without application from the registered person)
cancel the registration if a registered person fails to file his tax return for a
consecutive period of six (6) months. However, before such cancellation he will
have to give an opportunity of being heard to the registered person.
Federal Excise Duty
y Dutiable Goods [2(8b)]
y u means all excisable goods specified in
the First Schedule except those which are exempt u/s
16 of the Federal Excise Act, 2005.
y Dutiable Supply [2(8c)]
y u S means a supply of dutiable goods
made by a manufacturer. Supply of exempt goods shall
not be dutiable supply.
y Dutiable Services [2(8d)]
y u means all excisable services specified
in the First Schedule except those which are exempt u/s
16 of the Federal Excise Act, 2005.
ICMAP-STAGE 4, BUSINESS TAXATION REFRESHER

From the desk of SOHAIL MERCHANT

`

For complete and thorough coaching of Accounting and Business




Page 42
FED
y Franchise [2(12A)]
y l means an authority given by a franchiser under which
the franchisee is contractually or otherwise granted any right to
produce, manufacture, sell or trade in or do any other business
activity in respect of goods or to provide service or to undertake
any process identified with franchiser against a fee or
consideration including royalty or technical fee, whether or not a
trade mark, service mark, trade name, logo, brand name or any
such representation or symbol, as the case may be, is involved.
y Goods [2(13)]
y C means the goods leviable to excise duty under the Federal
Excise Act, 2005 or as specified in the First Schedule and includes
the goods manufactured or produced in non-tariff area and
brought into the tariff area for use or consumption.
y Non-tariff area means Azad Jammu and Kashmir, Northern Areas
and such other territories or areas to which the Federal Excise Act,
2005 does not apply. [2(17)]
y Tariff area means the areas other than the non-tariff area. [2(24)]
FED
yManufacture [2(16)]
y General Meanings
y It means and includes any process incidental or ancillary to the
completion of a manufactured product and any process of re-
manufacture, remaking, reconditioning or repair and the process of
packing or re-packing such product.
y Meanings in Relation to Tobacco
y M
y Cigarettes,
y Cigars,
y Cheroots (cigars with both end open),
y Biris,
y Cigarette, pipe or hookah tobacco,
y Chewing tobacco, or
y Snuff (powdered tobacco taken into the nose by snuffing).
ICMAP-STAGE 4, BUSINESS TAXATION REFRESHER From the desk of SOHAIL MERCHANT
`
For complete and thorough coaching of Accounting and Business




Page 43
y
Scope of FED
y The federal excise duty is levied and collected on excisable
goods and services of the following categories: [3(1)]
y The goods which are produced or manufactured in Pakistan.
y The goods which are imported into Pakistan.
y The goods which are produced or manufactured in the non-tariff
areas and are brought to the tariff areas. (These goods are notified
by the Federal Government in the official Gazette)
y The services provided in Pakistan including the services originated
outside but rendered in Pakistan.
y The excise duty is levied and collected at the rate of 15% ad
Valorem. However, the goods and services specified in the
First Schedule to the Federal Excise Act shall be charged to
duty at such rates as are specified against each goods and
service.
y Excise Duty on Imported Goods [3(2)]
y Where any excisable goods are imported into Pakistan, the excise
duty in respect of such goods shall be levied and collected in the
same manner and at the same time as if it were a customs duty
payable under the Customs Act, 1969. Under this case all the
provisions of that Act shall apply.
FED
Basis of the Duty [3(1) & (3)]
The duty may be charged on any of the following
basis:
Percentage to the value of the goods or services
(ad valorem);
Percentage to the retail price of the goods or
services;
Production capacity of the plant, machinery,
installations, etc.; or
Fixed duty.
ICMAP-STAGE 4, BUSINESS TAXATION REFRESHER

From the desk of SOHAIL MERCHANT

`

For complete and thorough coaching of Accounting and Business




Page 44
y
Value For The Purposes Of Duty [12]
y The Federal excise duty is generally levied on the value of the goods or
services. Under such a case, the determination of the value of goods or
services is very important. Legal provisions in this regard are discussed
below:
y Value of Goods [12(1), (3), (4) & (5)]
y Where any goods are liable to duty under the Federal Excise Act at a rate
dependent on their value, duty shall be assessed and paid on the basis of
value as determined under section 2(46) of Sales Tax Act, 1990, excluding
the amount of duty payable on these goods.
y Where the excise duty is chargeable on the goods at the import stage,
the value for excise duty shall be the value as determined under section
25 of the Customs Act, 1969 for customs duty or increased by the
amount of customs duty payable in respect of such goods. [12 (3)]
y Where the excise duty on any goods is chargeable on the basis of Retail
Price the value of such goods shall be the retail price fixed by the
manufacturer at which a particular brand or variety of such goods should
be sold to the general body of consumers. This price shall include all
duties, charges and taxes other than the sales tax. Where more than one
retail price is fixed for the same brand or variety, the highest of such price
shall be taken for charging the excise duty. [12(4)]
FED- Value of Goods
y Where the duty is chargeable on the basis of retail price the retail price
shall be legibly, prominently and indelibly indicated on each good, packet,
container, package, cover or label of such goods. The Board has the
authority to direct otherwise. [12(4)]
y The Board is empowered to specify any goods or class of goods on which
duty, in case of local production, is payable on retail price. In case of import
of such goods the duty shall be charge in the like manner as is payable in
case of goods locally produced. [Proviso to 12(4)]
y The Board may fix the minimum price of any goods or class of goods for the
purpose of levying and collecting the excise duty. Where the price at which
goods or class of goods is sold is higher than the price fixed by the Board,
the duty shall be charged and levied at such higher price. However, the
Board may also direct otherwise. [12(5)]
y Value of Services [12(2)]
y The value of services, in a case where excise duty is chargeable on the
charges for the services, shall be the total amount of charges for the
services, including the ancillary facilities or utilities, if any.
y Where any such services are provided free of charge or at a concessional
rate, the duty shall be charged on the amount which would have been
charged if the services, etc., had not been provided free of charge or at a
concessional rate.
ICMAP-STAGE 4, BUSINESS TAXATION REFRESHER

From the desk of SOHAIL MERCHANT

`

For complete and thorough coaching of Accounting and Business




Page 45

SALES TAX
By: Kashif Nawaz Jakhar 0331-4791167 Page 3
SALES TAX

Gross Sales Tax @ 17 %
Example :

Sr. No. Particular Value GST (%) GST Value Increase GST
1 Fans 50,000 17% 8500 58500
2 AC 50,000 17% 8500 58500
100,000 17,000 117,000

NOTE :
Tax on sale = Output Tax Collect
Tax on Purchase = Input Tax Paid




Income Tax Direct Tax
Sales Tax Indirect Tax
Paid by other person

Note :
Except consumer every person pays two times tax.

e.g.
Mr. Kashif purchases goods from Mr. Rameez. Kashif paid
input tax to Mr. Rameez. He also sold goods to Mr. Haseeb and
collect output tax . which he paid into NBP.
so,

Kashif Rameez



NBP
Point to Remember
O
u
t
p
u
t

T
a
x

p
a
i
d

Input Tax paid
SALES TAX
By: Kashif Nawaz Jakhar 0331-4791167 Page 4
Example :

Mr. Y purchased good worth Rs. 100,000 from Mr. Z. He
paid input Tax Rs. 17000. He sold these goods to Mr. X for
Rs. 120,000 and collect output tax Rs. 20,400 .
Calculation of Tax liability of Mr. Y











Calculation Of amount received and paid.


Purchase Price = 100,000 Sale Price = 120,000
Tax ( 17% ) = 17,000 Tax ( 17% ) = 20,400
Amount to be Paid 117,000 Amount to be received 140,400


Calculation of Tax Liability :

Sale ( out put Tax ) = 20,400
Purchase ( input Tax ) = (17,000)
Tax Liability 3400









Z Y X
Purchase. Rs.100,000 Sale. Rs. 120,000
Amount Paid. Rs.117,000
to Z
Amount Received.
Rs.140,400 From X
20,000 X 17% = 3400


= Sale - purchase
= 120,000-100,000
= 20,000
SALES TAX
By: Kashif Nawaz Jakhar 0331-4791167 Page 5
Example : { sale by one person is the purchase of other person }


Persons Purchase

Input Tax
( 17% )
Sale Output Tax
( collect )
NBP
Importer
(X)
10,000 1700 12,000 2040 1700
2040
Wholesaler
( Y )
12,000 2040 15,000 2550 510
Retailer
( Z )
15,000 2550 20,000 3400 850
Consumer
( K )
20,000 3400
3400

EXAM FOCUS POINT :
There are two possible ways in which the
data will be given in Exam.
Then calculation of sales Tax ( assume figures )

Excluded sales Tax = 20,000 X 17% = 3400
Inclusive of Sales tax = 20,000 X 0.17/1.17 = 2906
















Assume Custom duty is included
SALES TAX
By: Kashif Nawaz Jakhar 0331-4791167 Page 6
!!!!!!!! IMPORTANT RULES OF SALES TAX !!!!!!


1. Sales Tax Liability = Output Tax input Tax
2. Tax Fraction Formula = Amount X 17 / 117
3. The Person must be a registered person
4. He has paid the Tax
5. The activities must be Taxable
6. Goods Must be taxable
7. Input tax can be claimed for last 6 tax periods
8. Input tax on fixed assets shall be claimed in 12 tax periods
9. He holds invoice of Tax
10. No input tax on the fake invoice
11. No input tax on vehicle ( otherwise as an inventory )
12. Input Tax can be claimed on accrual basis
( must be paid with in 180 days )
13. All payments > Rs. 50,000 must be through banking channel
14. No input Tax on exempt goods
15. Common Tax shall be allowed in proportion to sale
16. Input Tax is restricted upto 90% of output Tax
17. Tax also be payable on drawings , taken up by the employees
18. No input Tax , if Tax is not paid by the supplier .
( To whom you have paid the Tax )












SALES TAX
By: Kashif Nawaz Jakhar 0331-4791167 Page 7
Explanation of the Rules :

Person:
A person can be a
x Individual
x AOP
x Company
But he must inform that from which category he belongs.
Categories :
x Retailer
x importer
x Exporter
x wholesaler
x Manufacturer
Registered Person: ( Ref. book page # 616 )
how to get register
Rules For Registration :

1. Compulsory Registration
2. Voluntary Registration
1. Compulsory Registration : ( Ref. book page # 617 )
a) For the followings, registration is compulsory
without any condition.
o Importer
o Distributor
o Wholesaler
b) For Manufacturer & Retailer :

Voluntarily:
If turnover of 12 months is less
than (<) 5 million.
Compulsory:
If turnover of 12 months exceeds
(>) 5 million.


SALES TAX
By: Kashif Nawaz Jakhar 0331-4791167 Page 8
2. Voluntary Registration :
If a person wants , he can be a
registered person.
Note :
A unregistered person can't claim or collect the tax.
He can't issue the invoice. 8ut he can pay the sales Tax.
Process of Registration :
If a person wants to be a person
the he should follow the following procedure.
Assume he lives in Lahore,
Application for registration will be
submitted to LRO ( local registration office)
LRO will send this application to CRO
( central registration office ). They can
directly inform the person about
registration or rejection.



aik person, aik waqt main aik say ziada jaga
per register ho sakta hai. means
As a wholesaler
As a importer etc.
usy for each category separately , sales tax number issue kiye ja
sakty hain.
aor aik single Sales tax number for all categories bhi issue kia ja
sakta hai .
x De-registration :
x Black listing:
A person will be de-recognized on the
following basis and he will be added in black list on
the web.
No tax submit
Issue fake invoice
x Voluntarily .

A person can claim only if he already paid the tax.
Note. ( in Urdu )
SALES TAX
By: Kashif Nawaz Jakhar 0331-4791167 Page 9
Taxable Activities: ( Ref. book page # 603 )
e.g.
Bought Tissue box for
x car
x office use only
then it is not a taxable activity , because these are
personal nature .
Bought Tissue box for
x Sale
x For use in manufacturing process.
it's a taxable activity.

Goods must be taxable: ( Ref. book page # 601 )
Land , building , Intangible asset ,
cash and shares are not goods.

GOOGS









Valid invoice of Tax: ( Ref. book page # 604 )
An invoice must not be
o Fake Invoice
o Issued by a black listed
Information given in Invoice ( Ref. book page # 604 )
o Name , address , and registration number of
supplier and recipient
o Date of issue
o Description and quantity of goods
o value exclusive of Tax
o Amount of sales Tax
Taxable Goods
Zero Rated
( 0% )
No Tax
Exempt Goods
17%
SALES TAX
By: Kashif Nawaz Jakhar 0331-4791167 Page 10
Fixed Assets :
The fixes asset must be used in the
production of taxable goods.
Exempt Goods: ( Ref. book page # 607 )
o Live animals
o Holy Quran and other holy books
o computer software
o Imported samples






























SALES TAX
By: Kashif Nawaz Jakhar 0331-4791167 Page 11






Example : ( Assumed Figures )

Supplies Amount Common Input Tax
Taxable
1000 840
Zero rated
500 420
Exempt
1000 840
2500 2100

Adjustment :
Purchases from unregistered person Rs.500 are equally
contributed to supplies .

Solution :
We can't claim tax on these purchases. So we must
deduct it from our sales.

Supplies Amount Common Input Tax
Taxable (1000-200)
800 840
Zero rate (500-100)
400 420
Exempt (1000-200)
800 840
2000 2100




In both cases Total of common input tax will be same.







CONCEPT
About
JRRGVSXUFKDVHGIURPXQUHJLVWHUHGVXSSOLHUV
are exclusively used for making Taxable supplies
SALES TAX
By: Kashif Nawaz Jakhar 0331-4791167 Page 12

From sale only the Sale price of , purchases from un-
registered person will be deduct.

Question :
Purchases from unregistered person of Rs.400 whose sale
price is Rs.500. These goods were used in the Taxable supplies.
Common input Tax is Rs.2100

Supplies Amount
Taxable
1000
Zero rated
500
Exempt
1000
2500
Requirement:
Apportionment of common input Tax.

Solution :
Apportionment of common input Tax

Supplies Amount Common Input Tax
Taxable (1000-500)
500 525
Zero rate
500 525
Exempt
1000 1050
2000 2100

Apportionment of Common input Tax.

Taxable supplies =


Zero Rated =


Exempt Supplies =




NOTE
500
2000
X 2100 = 525
500
2000
X 2100 = 525
1000
2000
X 2100 = 1050
SALES TAX
By: Kashif Nawaz Jakhar 0331-4791167 Page 13

If sale price of purchases from unregistered person is not
given.

EXAMPLE :

Purchase price of goods from unregistered person is Rs.400 and
these goods were used in the taxable supplies. Purchases from
registered person is Rs. 200. Common input tax is Rs. 2100.

Supplies Amount
Taxable
1000
Zero rated
500
Exempt
1000
2500
Requirement:
Apportionment of common input Tax.


Solution :
IF purchase price not given then the following procedure
should be adopted.
I. Calculate the gross profit ratio
II. Apply the ratio on purchases and calculate the sale
price of the purchases.













Note
NOTE
Formulas for calculating the gross profit ratio.

S = C + S X %
OR
C = C + C X %
where ,
S = Total sales of the Company
C = Total cost ( purchases ) of the company
SALES TAX
By: Kashif Nawaz Jakhar 0331-4791167 Page 14

( W-I ) Calculation of margin on sale :

S = C + S X %
assume : % = X

2500 = 400 + 2500X
2500X = 2500 - 400
2500X = 2100
X = 0.84% or 0.0084

( W-II ) Calculation of the sale prices of the purchases from
unregistered person.

S = C + S X %
x = 600 + 0.0084x
0.9916x = 600
x = 605
So now ,
= Taxable supplies - Sale price of purchases from unregistered person
= 1000 - 605
= 395

Apportionment of common input Tax

Supplies Amount Common Input Tax
Taxable (1000-605)
395 438
Zero rate
500 554
Exempt
1000 1108
1895 2100








SALES TAX
By: Kashif Nawaz Jakhar 0331-4791167 Page 15

!!!!!!!!!!! General format of Sales Tax Numerical !!!!!!!!!!

Computation of Tax Liability

Output Tax Rs.(000)
Supplies to registered person ( xxx X 17% ) xxxx
Supplies to un-registered person ( xxx X 17% ) xxxx
xxxx
Input Tax
Common input tax ( w-1 ) xxxx
or
90% of out put Tax xxxx
(xxxx)
Tax Liability xxxx

Tax Refundable ( w-1 ) = xxxx
Note :
if the 90% of output tax is less than the common input tax
then the balance will be carry forward.
e.g.
common input tax = 1000
90% of output tax = 800
Balance c/f = 200 it would be the input tax of the next month

Working :
( W-I ) Apportionment of common input tax.

Supplies Rs.(000) Common input Tax
Rs.(000)
Taxable xxx xxx
Zero Rated xxx xxx ( refundable )
Exempt xxx xxx ( not refundable )
xxxx xxxx

( W-I ) Common input Tax or Residual Tax.

Net purchases from registered person X 17% = xxxx

Lower

=
Taxable supplies
Total supplies

X total input tax
SALES TAX
By: Kashif Nawaz Jakhar 0331-4791167 Page 16

Debit and Credit Note( Ref. book page # 612 )

Example :
In June Mr. X sales goods to Mr. Y worth Rs.100,000
and receive Rs.117,000( where 17000 is sales Tax ) . But in July Mr. Y
return goods worth Rs.50,000.
So Mr. X will return Rs. 58,500 { 50,000 + ( 17000 X 50% ) }.

Information given:
Mr. X Mr. Y
Issue credit note ( sales return ) Issue Debit note ( purchase return )

Output Tax = Rs. 100,000 Output Tax = Rs. 200,000
Input Tax = Rs ( 50,000 ) Input Tax = Rs ( 80,000 )


Requirement :
Calculate Tax payable by both persons.

Solution :

Mr. X Mr. Y

Output Tax = 100,000 Output Tax = 200,000
less: credit note = ( 8,500 )
91,500 Input Tax = ( 80,000 )
Less: input Tax = (50,000) Debit Note = 8,500 .
Tax Payable = 41,500 ( 71,500 )

Tax Payable = 128,500








Difference is
tax payable
SALES TAX
By: Kashif Nawaz Jakhar 0331-4791167 Page 17

Theoretical Portion of Sales Tax

General terms :

1. Sales Tax is a multi stage tax : ( Ref. book page # 598 )
The Sales tax is a multistage
tax payable on the value of :
Taxable supplies by a registered person in respect of any
taxable activity carried on by him ;
Goods imported in Pakistan ; and
Specified taxable services
2. Who is liable to pay Sales Tax : ( Ref. book page # 599 )
Liability to pay the sales tax
to the sales tax department shall be :
of the person making the supply , in case of supply of
goods ;
of the person importing the goods , in case of goods
imported in Pakistan; and
of the person providing taxable services.
3. When to pay Sales Tax : ( Ref. book page # 599 )
Sales tax shall be paid at the time
of:
payment of custom duty in case of import of goods ;
Filing of sales tax return in case of supplies made or
services provided in Pakistan.

4. Definition of Cottage industry : ( Ref. book page # 599 )
Means a manufacturer whose
annual taxable turnover during the last 12 months , ending any tax
period , does not exceeds Rs.5 million or whose annual utility
( electricity , gas, and telephone ) bills during the last 12 months ,
ending any tax period, do not exceed Rs.700,000
In Short:

I. Taxable turnover of last 12 months < Rs.5 million
II. Utility bills of last 12 months < Rs. 700,000


SALES TAX
By: Kashif Nawaz Jakhar 0331-4791167 Page 18





Importer / wholesaler /
Distributor/ Retailer
Sales tax @ 17% on value of taxable
supplies
Total turnover upto Rs.5 million during
the last 12 months.
Exempt supplies as per 6
th
schedule.
Total turnover exceeds Rs.5 million
during the last 12 months.
Fixed sales tax as per special
procedure rule
Manufacturer

Cottage industry Exempt supplies as per 6
th
schedule.
Other than Cottage industry Sales tax @ 17% on value of taxable
supplies.


IMPORTANT DEFINITIONS:

1. Arrears: ( Ref. book page # 600 )
Means unpaid sales Tax including;
x additional tax
x default surcharge
x fine
x penalty
x fee
x or any other sum under the sales tax laws at
any particular day.
2. Defaulter: ( Ref. book page # 601 )
Means a person who fails to pay the arrears.
Person is a ;
x proprietor
x a partner in case of a firm
x a director in case of a company
x and also includes a guarantor or successor



Exemptions provided to small manufacturer and small retailer
SALES TAX
By: Kashif Nawaz Jakhar 0331-4791167 Page 19

3. Distributor: ( Ref. book page # 601 )
Means a person appointed by a manufacturer,
importer , or any other person for a specified area to purchase
goods from him for further supply and includes a person who is also
engaged in supply as a wholesaler or retailer.
4. Due Date: ( Ref. book page # 601 )
In relation to the furnishing of a return means the 15
th

day of the month , following the , end of the tax period or such
other date as the FBR may specify.
5. Goods: ( Ref. book page # 601 )
Includes every movable property other than actionable
claim , money, stocks and shares and securities .
6. Input Tax: ( Ref. book page # 601 )
In relation to registered person means , the tax
levied on :
x supply of goods to the person
x Goods imported and cleared under the custom act.
x taxable services under the provincial ordinance
rendered to the person
x Federal excise duty in sales tax mode on goods or
services to the person.
7. Manufacture or Produce: ( Ref. book page # 601 )
Includes :
o any process in which an article is converted into
another distinct article
o any process in which an article is so changed or
reshaped that it becomes capable of being put to
use differently
o any process incidental to the completion of a
manufactured product
8. Manufacturer or Producer: ( Ref. book page # 602 )
Means a person who engages in the
manufacture or production of goods whether or not the raw
materials are owned by him.


SALES TAX
By: Kashif Nawaz Jakhar 0331-4791167 Page 20
9. Open market price: ( Ref. book page # 602 )
Consideration in money which that supply
or similar supply would generally fetch in an open market .
10. Output Tax : ( Ref. book page # 602 )
In relation to registered person means , the tax
levied on :
x supply of goods by the person
x taxable services under the provincial ordinance
rendered by the person
x Federal excise duty in sales tax mode on goods or
services by the person.

11. Sales Tax: ( Ref. book page # 603 )
Means;
x The tax ,
x additional tax,
x default surcharge,
x fine ,
x penalty,
x fee ,
x or any other sum under the sales tax laws.

12. Supply: ( Ref. book page # 603 )
Means the sale or other transfer of the right to dispose
off goods as owner, including such sale or transfer under a hire
purchase agreement , and also includes :
a. putting to private , business or non business
use of goods manufactured in the course of
taxable activity other than those of making a
taxable supply.
b. auction or disposal of goods to satisfy debt
owed by a person
c. possession of taxable goods held immediately
before a person ceases to be a registered
person

SALES TAX
By: Kashif Nawaz Jakhar 0331-4791167 Page 21
13. Taxable Activity: ( Ref. book page # 603 )
means any economic activity whether or not
for profit , and includes ;
I. any activity in the form of business , trade or
manufacture ;
II. supply of goods or providing of services to
another person;
III. a one-off adventure or concern in the nature of
a trade ; and
IV. any thing done or undertaken during the
commencement or termination of the economic
activity
14. Tax Fraction ( Formula ): ( Ref. book page # 603 )

% of sales Tax .
% of sales Tax + 100

15. Taxable Supplies : ( Ref. book page # 603 )
A supply of taxable goods by an;
x importer
x manufacturer
x wholesaler / dealer / distributor
x or retailer including zero rated supply
but excluding exempt goods.

16. Tax period: ( Ref. book page # 604 )
One month or other period as the Federal
government may specify .
17. Time of Supply : ( Ref. book page # 604 )
Time of supply depend upon different situations :
Situations Time of Supply
Normal Time at which the goods are delivered or made
available to the recipient of the goods
Supply under hire purchase
agreement
Date of agreement
Services Time at which services are provided

X Value
SALES TAX
By: Kashif Nawaz Jakhar 0331-4791167 Page 22

18. Value of supply : ( Ref. book page # 604 )
Value of taxable supplies is the consideration
in money including all federal and provincial duties which the
supplier receives in respect of the supply excluding the amount of
sales tax.

SR. no.
Situation Value of supply
1 Consideration is partly or fully in kind Open market price
excluding sales tax
2 Sale is made on installment basis
where the price includes markup on
surcharge
Open market price
excluding sales tax.
[ mark up or surcharge
included in credit sales or
sale on installment basis is
not subject to sales tax ]
3 Supply between associated persons Value as in the normal case
or open market price
excluding sales tax
whichever is higher
4 In case of imported goods Value determined under
the custom act including
custom and excise duty
levied thereon
5 If there is a reason to believe that the
value is under declared in the tax
invoice
Value determined by the
valuation committee
comprising representative
of trade and the sales tax
department
6 Special nature of transaction and
ascertainment of value is difficult
Open market price
excluding sales tax
7 Exempt goods supplied to a registered
person for processing
Market price excluding
sales tax

19. Tax rates: ( Ref. book page # 605 )
Sales Tax rate is 17%. however , the Federal
Government has power to fix a lower or higher rate on specified
items.
22% on various items such as jelly, plastic waste,
paper and aluminum
19.5% on various items such as certain steel items .

SALES TAX
By: Kashif Nawaz Jakhar 0331-4791167 Page 23

20. Zero Rates Supplies : ( Ref. book page # 606 )
Goods falling under this category are
chargeable to sales tax @ 0%.
Examples of items under this category :
o Supply to diplomats , diplomatic mission and
privileged persons
o supply of raw material
o supply to exporters under duty and tax
remission rules ( DTRE )
o Supplies against international tenders
Some more examples are given at book page 606.

21. Exempt Supplies: ( Ref. book page # 607 )
Certain imports and supply of goods falling
under this category are outside the scope of sales tax and there for
not subject to sales tax.
Examples of items under this category :
o Live animals
o Agriculture produce
o Holy Quran
o computer software
o imported samples
Some more examples are given at book page 606.

22. Debit and credit Note and destruction of goods;
( Ref. book page # 612 ) Where a
registered person has issued a tax invoice and the tax return or tax
invoice needs to be modified as a result of :
o cancellation of supply ; or
o return of goods ; or
o change in the nature of supply ; or
o change in the value of supply.
With in 180 days , then the registered person may issue a debit/
credit note indicating specified information including reason for the
issuance of debit/ credit note and make adjustments accordingly.
[ Period of 180 days may be extended for any special person ]

SALES TAX
By: Kashif Nawaz Jakhar 0331-4791167 Page 24
23. Drawing of Samples : ( Ref. book page # 616)
For the purpose of determining sales tax
liability or for the purpose of establishing value of any taxable goods,
an authorized officer can take minimum quantity of goods or raw
materials against a proper receipt, a copy of which shall be kept in
record by the registered person and the commissioner .




Important definitions are giving in the book at page no.600
Notes on income tax laws
Tax Year 2012
Muhammad Ovais, Deloitte 13th MFC


1

GENERAL
Heads of income (Section 11) Amount
Salary xxxx
Income from Property xxxx
Income from Business xxxx
Capital Gains Xxxx
Income from Other Sources Xxxx
Total Income (Section 10) Xxxx
Less Deductible allowances:
Zakat (Section 60) xxxx
WorkeUV:HOIDUH)XQG(Section 60A) xxxx
:RUNHUV3DUWLFLSDWLRQ)XQG(Section 60B) xxxx
Taxable Income (Section 9) xxxx

For the purpose of imposition of tax and the computation of total income, all incomes chargeable to
tax shall be classified under the above mentioned heads of income.

Furthermore, for the computation of income of a resident person, the considerations shall be given to
the amounts that are Pakistan source income and the amounts that are foreign source income. And for
the determination of income of a non resident person, only the amounts that are Pakistan source
income are taken into account for the computation of taxable income.

:RUNHUV:HOIDUH)XQG (Section 60A):
It is payable @ 2% of the taxable income. It should be added back to accounting profit and deducted
from total income to arrive at taxable income.

For the purpose of above, 'taxable income' means:
x Where return of income is required to be filed, accounting profit before tax or declared
income as per return of income, whichever is higher
x Where return of income is not required to be filed, accounting profit before tax or 4% of the
receipts as per statement of FTR u/s 115, whichever is higher

Residential Status of Persons:
For the purpose of determining the residential status of a person, the following rules mentioned in the
section 81-84 are considered along with Rule 14 of the Income Tax Rules, 2002. The principles for
determination of residential status are as under:

Individual: if he is present in Pakistan for 183 days or more in a tax year or he is an employee of
federal or provincial government posted abroad for performing his duties (Section 82)

Rule 14 of the Income Tax Rules, 2002: Rules for determination of physical stay in Pakistan of an
individual:

A part of a day that an individual is present in Pakistan counts as a whole day of such presence except
where the person is present in Pakistan solely by the reason of being in transit i.e. in a journey
between two different places in Pakistan does not count as a day present in Pakistan.
Notes on income tax laws
Tax Year 2012
Muhammad Ovais, Deloitte 13th MFC


2

The following days count as a whole day of such presence, namely:
A day of arrival in/departure from Pakistan A day that an individual's activity in Pakistan
is interrupted because of a strike, lock-out or
delay in receipt of supplies

Public holiday

Day of leave A holiday spent by the individual

Tax Rates: Mr. A Mr. B
Taxable salary 500,000 420,000
Capital gain 450,000 470,000
Taxable income 950,000 890,000

Taxable Salary 500,000 = 52% 420,000 = 47%
Taxable Income 950,000 890,000

Salaried Case Non-Salaried Case

Tax rates for individuals have been prescribed in the 1
st
schedule with respect to salaried and non
salaried case:
x Where taxable salary exceed 50% of the taxable income = Salaried Case
x Where taxable salary is less than 50% of the taxable income = Non Salaried Case

The basic exemption limit for the taxable income is enhanced from Rs. 300,000 to Rs. 350,000.
However individual taxpayers', whose normal income is between Ps. J00,000 to Rs. 350,000, shall be
required to file return of income and statement for the purposes of documentation.

Marginal Relief: provided that where the total income of a tax payer marginally exceeds the maximum
slab limit, the income tax payable shall be the tax payable on the maximum of that slab plus the
amount equal to the following as the case may be: (benefit not available to non-salaried case)

Rate shall be applied to the excess of the maximum in the slab Where total income:
20% Does not exceed 500,000
30% Does not exceed 1,050,000
40% Does not exceed 2,000,000
50% Does not exceed 4,450,000
60% Exceeds 4,450,000

Tax shall be calculated as per the slab rates or marginal relief formula, whichever is lower (Circular 6
of 2008)


Notes on income tax laws
Tax Year 2012
Muhammad Ovais, Deloitte 13th MFC


3

Company: if it is incorporated under any law in force in Pakistan, the control and management of
affairs of the company is situated wholly in Pakistan at any time in a year or it is a Provincial or Local
government (Section 83). Company (other than small company which shall pay tax @ 25%) is taxable at
35% on its taxable income for the tax year 2007 and onwards. (1
st
schedule)

AOP (includes partnership and joint venture): if the control and management of the affairs is
situated wholly or partly in Pakistan at any time in the year (Section 84). AOP is taxable at 25% on its
taxable income for the tax year 2010 and onwards. (1
st
schedule)
Scope of Taxable Income:
1. Foreign Source Income of a short-term resident individual: (Section 50)
Foreign source income of a resident individual who is a resident solely by reason of
employment and is present in Pakistan for a period not exceeding 3 years shall be
exempt provided that any foreign source income is not brought or received into
Pakistan.

Such exemption shall not apply to an income derived by the person through his business
established in Pakistan.

2. Foreign source income of returning expatriates: (Section 51)
Foreign source income of a resident individual who is resident in current tax year but
was non-resident in the preceding 4 tax years shall be exempt in the current tax year
and in the following tax year.

Tax Year: (Section 74)

Normal: July 01 - June 30
Special: Year ending other than June 30
Transitional: Period between the ends of last tax year till the date of commencement of changed tax
year.

A person using a normal or special tax year may apply to the Commissioner to allow him to use a twelve
months' period other than special or normal tax year and the Commissioner shall grant permission only
if the person has shown a compelling need to use a special or normal tax year as the case may be.


Notes on income tax laws
Tax Year 2012
Muhammad Ovais, Deloitte 13th MFC


4

SALARY

Salary is taxable on receipt basis (see Miscellaneous section for definition of receipt) i.e. any salary
received by an employee in a tax year shall be chargeable to tax. However, salary paid by a private
company to its employee in arrears may be taxable on accrual basis if the commissioner is of the view
that the payment of salary was deferred. (Section 110)

Salary means any amount received by an employee from any employment, whether of a revenue or
capital nature, and includes:

1. Pay, wages or other remuneration provided to an employee

2. Perquisite (Section 13) (Rule 3-6 of Income Tax Rules, 2002)

3. Allowances including cost of living, rent, utilities, education, entertainment or travel
allowance excluding any amount expended in the performance of duties of employment

4. Reimbursement of any expenditure incurred by the employee, other than expenditure incurred
solely in the performance of duties of employment

5. Profits in lieu of salary or wages including:
a. Consideration for an employee agreement to enter into an employment relationship, to
any conditions of employment or to a restrictive covenant to any past, present or
future employment
b. Amount received from a Provident Fund
c. Amount paid on termination of employment whether on voluntary basis or under any
agreement [Section 12 (6) + (8)]

6. Pension or annuity

7. Employee share Scheme (Section 14)

8. Tax paid by the employer on employee's salary (Tax on Tax)

An amount or perquisite shall be treated as received by an employee from any employment regardless
of whether the amount or perquisite is paid or provided:
x 8y the employee's employer, an associate of the employer, or by a third party under an
arrangement with the employer or an associate of the employer;
x By a past employer or a prospective employer; or
x To the employee or to an associate of the employee or to a third party under an agreement
with the employee or an associate of the employee.

Where salary is paid to an employee in arrears and as a result his income is chargeable to higher rate of
tax than that would have been applicable if the amount had been paid in the tax year in which he was
entitled to receive, he may by a notice in writing to the commissioner by the due date for furnishing
employees return of income or employer certificate, elect for the amount to be taxed at the rates that
would have been applicable if the amount had been paid in the tax year in which he was entitled to
receive.

Notes on income tax laws
Tax Year 2012
Muhammad Ovais, Deloitte 13th MFC


5

Tax shall be deducted by the employer at the time of payment of salary, at the average rate of tax
after making adjustments of tax withheld from employee under other heads and admissible tax credits
during the year. (Section 149)

Salary shall be Pakistan source income where the salary is: (Section 101)
x Received from any employment exercised in Pakistan, wherever paid; or
x Paid by or on behalf of FG, PG or LG in Pakistan, wherever the employment is exercised.

Taxability of the above mentioned items is as under:

2. Perquisite: (Section 13)
a. Motor Vehicle provided by an employer wholly or partly for private use, the amount
chargeable to tax under the head salary shall include: (Rule 5 of Income Tax Rules,
2002)
i.
Partly for personal and official use 5% of the cost of vehicle to the
employer or FMV of motor vehicle at
the commencement of lease
For personal use only 10% of the cost of vehicle to the
employer or FMV of motor vehicle at
the commencement of lease

b. Services of housekeeper, gardener driver, or other domestic assistant, the amount
chargeable to tax under the head salary shall include the amount of total salary paid to
the domestic assistants as reduced by any payment made to the employer for such
services
c. Utilities, the amount chargeable to tax under the head salary shall include the FMV of
utilities as reduced by any payment made to the employer for such utilities

d. Interest free loan or a loan made at lower than benchmark rate by the employer on or
after 1.7.2002, the amount chargeable to tax under the head salary shall include the
profit on loan computed at benchmark rate or the difference between the amount of
profit paid and the amount of profit on loan computed at benchmark rate (for tax year
2003 is 5% and increased by 1% for each successive year)
i. Provided that this shall not apply where such benefit is extended by the
employer due to waiver of interest by such employee on his accounts
maintained with the employer

e. For the purpose of clause'd' above, where the employee uses the loan referred in that
clause, wholly or partly for the acquisition of any asset or property producing income
chargeable to tax under any head of income, the employee shall be treated as having
paid an amount as profit equal to the benchmark rate on the loan or that part of the
loan used to acquire the asset or property. (Indirectly by contributing to the national
economy in the form of tax on taxable income).

f. Any obligation waived of an employee owed to the employer

g. Any obligation paid by the employer of an employee owed to another person

Notes on income tax laws
Tax Year 2012
Muhammad Ovais, Deloitte 13th MFC


6

h. Difference between the FMV of any property transferred or service provided to the
employee by the employer and the amount of any payment made by the employee in
this respect

i. Accommodation or housing, the amount chargeable to tax under the head salary shall
include: (Rule 4 of the Income Tax Rules, 2002)
i. Higher of the following:
1. Amount that would have been paid in case such accommodation was
not provided; and
2. 45% (30% in case of persons serving in stations in Mufasal areas) of the
MTS of the basic salary or the basic salary where there is no MTS.


5. Profits in Lieu of Salary or Wages: [Section 12 (6) + (8) ]
a. An employee who has received amount on termination of employment whether on
voluntary basis or under any agreement including compensation for redundancy or loss
of employment and golden hand shake payments, may elect for the amount to be taxed
at the last 3 year's average rate of tax computed as follows, provided that such
election shall be made by the due date for furnishing employees return of income or
employer certificate by a notice in writing to the commissioner:
i. Last J year's tax liability / Last J year's taxable income

7. Employee Share Schemes: (Section 14)
a. Right or option to acquire shares: Taxable under the head salary when the same are
disposed off as the difference between consideration received less cost of option or
right

b. Issue of Shares: Taxable under the head salary at the time of issue when shares are
issued without restriction as the difference between the FMV of shares issued at the
date of issue less any amount paid by the employee for shares and for the grant of right
or option to acquire shares.
i. Where shares are issued with restriction on transfers, no amount shall be
chargeable to tax until the employee has a free right to transfer the shares or
the time the employee disposes of the shares, whichever is earlier

ii. Where share are issued with restriction on transfers, the amount chargeable to
tax under the head salary includes the difference between the FMV of shares
issued at the date mentioned above less any amount paid by the employee for
shares and for the grant of right or option to acquire shares.

c. Gain on shares subsequently disposed off falls under the head capital gain and for this
purpose, cost of shares shall be the total of consideration given by the employee for
shares, option/right and the amount taxed under the head salary in this respect.





Notes on income tax laws
Tax Year 2012
Muhammad Ovais, Deloitte 13th MFC


7

Difference between bonus shares and shares issued under employee stock option scheme:
x Bonus shares are issued to shareholders in lieu of cash dividend and are not taxable at the time
of issue, as the same are excluded from the definition of income.
x Shares issued under employee stock option scheme are issued to employees as an employment
benefit and are taxable under the head salary.
x Option under employee stock option scheme is a capital asset as per the definition of capital
asset u/s 37(5). However, this is specially taxed under the head salary as per section 145 only
when it is disposed off.

8. Tax on Tax:

Important points to note:
x Tax rate may change due to tax borne by the employer. In this case, apply new slab rate from
the beginning if it appears that slab rate may change after adding tax borne by the employer.
x In case where the employer agrees to pay specified amount of tax and the excess is to be paid
by the employee, no grossing up formula is required. Specified amount of tax borne by the
employer is taxable in the normal manner.
x Where employee is required to pay specified amount of tax and any excess is to be paid by the
employer, only the amount in excess of the specified amount shall be grossed up.
x Employer may also agree to pay a certain percentage (%) of tax liability of the employee.

Example:
Basic salary 318,000
HRA 127,200
Company maintained car for both purposes costing 900,000
Medical allowance 31,800
Tax of 11,000 is to be deducted from salary and the balance is to be borne by the company.

Solution:

Basic salary 318,000
HRA 127,200
Company maintained car 45,000
Medical allowance 31,800
Less: 10% of basic salary (31,800)
Taxable income 490,200

Tax liability @ 3.5% 17,157

Tax liability under marginal relief:
450,000 * 2.5% = 11,250
40,200 * 20% = 8,040
Tax liability 19,290

Tax liability whichever is lower 17,157
Tax to be borne by the employer (17,157-11,000) = 6,157/96.5% = 6,380

Notes on income tax laws
Tax Year 2012
Muhammad Ovais, Deloitte 13th MFC


8

Taxable income 490,200
Tax borne by the employer 6,380
Total taxable income 469,580

Tax liability @ 3.5% 17,380
Tax paid by the employee 11,000
Tax borne by the employer 6,380

Example:
Basic salary 300,000
HRA 120,000
Entertainment allowance 20,000
Company maintained car for both purposes costing 900,000
Medical allowance 30,000
70% of the tax is to be borne by the company.
Solution:
Basic salary 300,000
HRA 120,000
Entertainment allowance 20,000
Company maintained car 45,000
Medical allowance 30,000
Less: 10% of basic salary (30,000)
Taxable income 485,000

Tax liability @ 3.5% 16,975
Tax liability under marginal relief:
450,000 * 2.5% = 11,250
35,000 * 20% = 7,000
Tax liability 18,250

Tax liability whichever is lower 16,975
Tax to be borne by the employer (16,975/97.55%) * 70% = 12,181

Taxable income 485,000
Tax borne by the employer 12,181
Total taxable income 497,181

Tax liability @ 3.5% 17,401
Tax borne by the employer (70%) (12,181)
Tax paid by the employee (30%) 5,220

Grossing up formulae: (16,975/97.55%) * 70% = 12,181
100% (3.5%) - 3.5% * 70% (70%) = 97.55%
Always the same, it
testifies the
correctness of the
solution.
Always the same, it
testifies the
correctness of the
solution.
Notes on income tax laws
Tax Year 2012
Muhammad Ovais, Deloitte 13th MFC


9

Exemptions from salary income: (2
nd
Schedule)
a) Pension received by the citizen of Pakistan from the former employee except where the person
continues to work for the same employer or an associate of the employer. Where a person
receives more than one pension, the exemption shall apply to higher of such pensions. (Clause
8)
i. For a person over 60 years of age, all such pensions are exempt irrespective of the
above mentioned conditions (Circular 28 of 1991)

b) Commutation of pension received from government or any pension scheme approved by the tax
authorities (Clause 12)

c) Gratuity received from approved gratuity fund is fully exempt. Gratuity received from
approved scheme and unapproved fund or scheme is exempt up to the following limits: (Clause
13)

i.
Approved gratuity scheme Exempt up to 200,000
Unapproved gratuity scheme/fund Exempt up to 75,000 or 50% of the
amount receivable whichever is lower Unapproved commutation of pension

ii. Exemption in respect of unapproved gratuity/commutation of pension shall not apply in
the following cases:
a) Any payment not received in Pakistan
b) Any payment received by a director of a company who is not a regular
employee of such company
c) Any payment received by a non-resident
d) Any gratuity received by an employee who has already received any gratuity
from the same or other employer.

d) Accumulated balance received from a recognized provident fund (Clause 23)
x However, there are limits, in excess of which the employer's annual contribution and
interest credited on the balance of the employee shall be deemed to be income
received by the employee: (Clause 3, Sixth Schedule)
x /LPLWRIHPSOR\HUVFRQWULEXWLRQ 100,000 or 1/10
th
of the basic salary whichever is
lower
x Limit on interest credited to employee balance: 1/3
rd
of basic salary or 16% on the
accumulated balance, whichever is lower.

e) Amount received from WPPF (Clause 26)

f) Any special allowance provided to meet the expenses incurred in performing of office duties
(Clause 39)

g) Perquisites: (Clause 53A)
x Free or concessional passage provided by transporters to its employee including airlines
x Free or subsidized food provided by hotels and restaurants to its employees during duty
hours
Notes on income tax laws
Tax Year 2012
Muhammad Ovais, Deloitte 13th MFC


10

x Free or subsidized education provided by an educational institution to the children of
employees
x Free or subsidized medical treatment provided by a hospital or clinic to its employees
x Any other perquisite for which the employer does not have to bear any marginal cost

h) Medical facility or the reimbursement received by an employee where such provision or
reimbursement is in accordance with the terms of employment provided that the NTN of the
hospital or clinic is provided and employer also certifies and attests the medical bills.

i) Medical allowance up to 10% of basic salary (The same is fully taxable if it is provided in
addition to the exempt medical facility provided by the employer)

j) Salary earned outside Pakistan by the citizen of Pakistan during the tax year in which he leaves
Pakistan shall be exempt if he leaves Pakistan during the tax year and remains abroad during that
tax year. [Section 51 (2)]

k) Foreign source salary received by a resident shall be exempt if the individual has paid foreign
income tax in respect of that salary or the employer has withheld income tax in respect of foreign
source salary and paid to the revenue authority of that foreign country in which the employment
was exercised. (Section 102)

Example: Provident Fund
Contribution by
Interest Income
Name Employee Employer
A 10,000 10,000 4,000
B 20,000 20,000 8,000
C 30,000 30,000 12,000

Balance of the fund = 120,000
Interest income 24,000 @ 20%
Dr: Cash 24,000
Cr: Mr. A 4,000
Cr: Mr. B 8,000
Cr: Mr. C 12,000

Situation relates to Unrecognized Provident Fund:
In 2011, Mr. A received Rs. 24,000 from PF. Taxability of the amount received is as under:
Employee contribution 10,000 (Already Taxed)
Employer contribution 10,000 (Taxable)
Interest income 4,000 [Taxable under the head salary u/s 12(2)(e)(iv)]

Situation relates to recognized Provident Fund:
In 2011, Mr. A received Rs. 24,000 from PF. Taxability of the amount received is as under:
Employee contribution 10,000 (Already Taxed)
Employer contribution 10,000 (Exempt)
Interest income 4,000 (Exempt)
Notes on income tax laws
Tax Year 2012
Muhammad Ovais, Deloitte 13th MFC


11

Example:
Basic salary 1,200,000
Dearness allowance 120,000
Employer's contribution to PPF: 120,000
Less: Rs. 100,000 or 10% of basic + DA
Whichever is lower: (100,000)
1,340,000
Interest credited @ 17% 476,000
Less: Interest @ 16% or 1/3
rd
of salary
Whichever is lower: (440,000)
Taxable salary 1,176,000

Temporary withdrawal: No tax treatment
Permanent withdrawal: Tax treatment is the same as in the case of amount received from PF on
termination of employment.


Notes on income tax laws
Tax Year 2012
Muhammad Ovais, Deloitte 13th MFC


12

INCOME FROM PROPERTY

Chargeable Rent includes:
Normal Rent (higher of actual or fair market rent)
Non adjustable amount in respect of building
Forfeited deposit
Signing amount

Actual rent or Fair Market Rent, whichever is higher is taxable on accrual basis

Exclusions from section 15 (Chargeable to tax under the head income from Other Sources):
x Rent in respect of lease of building together with plant and machinery
x Amount received for the provision of amenities, utilities and any other service connected with
renting of the building

Taxability of property income is as under:

For individual and AOP:
Gross amount of Rent Rate of Tax
Does not exceed 150,000 0%
Exceeds 150,000 but less than 400,000 5 % of the amount exceeding 150,000
Exceeds 400,000 but less than 1,000,000 12,500 + 7.5% of the amount exceeding 400,000
Exceeds 1,000,000 57,500 + 10% of the amount exceeding 1,000,000

For Company:
Gross amount of Rent Rate of Tax
Does not exceed 400,000 5 % of the gross amount
Exceeds 400,000 but less than 1,000,000 20,000 + 7.5% of the amount exceeding 400,000
Exceeds 1,000,000 65,000 + 10% of the amount exceeding 1,000,000

This section shall not apply in respect of a tax payer who is an individual or an AOP, derives income
chargeable to tax under this section not exceeding 150,000 and does not derive taxable income under
any other head.

Where any property is owned by two or more persons and their respective shares are defined and
ascertainable, such persons shall not be treated as an AOP in respect of that property, and the share of
each person is taxable in the hands of each co-owner separately. (Section 66)

Non adjustable amounts against the rent of a building received from a tenant in respect of rent
payable by him shall be chargeable to tax under the head income from property in the tax year in
which it was received and in the following nine tax years in equal proportion. Where the same is
refunded to the tenant on or before the expiry of 10 years, no amount therefore shall be chargeable to
tax in the year in which it is refunded subject to the following requirement: (Section 16)
x If the same property is lent out to the succeeding tenant, then any succeeding non
adjustable amount received from the succeeding tenant as reduced by such portion of
the earlier amount as was charged to tax shall be charged to tax in the year in which it
was received and in the following nine tax years in equal proportion.
Notes on income tax laws
Tax Year 2012
Muhammad Ovais, Deloitte 13th MFC


13


Tax Deduction at Source: (Section 155)
Following prescribed persons shall deduct tax at source when making payment on account of rent of
immoveable property:
x Federal, provincial or local government
x Company
x Nonprofit organization
x Diplomatic mission of a foreign state

Income from property is not taxable under FTR and no related expenses can be claimed against such
income.


Notes on income tax laws
Tax Year 2012
Muhammad Ovais, Deloitte 13th MFC


14

INCOME FROM BUSINESS

The following incomes of a person for a tax year shall be chargeable to tax under the head income
from business:
x Profits and gains of any business carried on by a person
x Income from the sale of goods or provision of services
x Income from hire or lease of tangible moveable property
x Management fee derived by a management company (including a modaraba management
company)
x FMV of any benefit derived by a person in the course of or by virtue of past, present of
prospective business relationship
x Profit on debt where the person's business is to derive such income (e.g. banks and financial
institutions)
x Lease Rentals earned by a scheduled bank, investment bank, DFI, modaraba or a leasing
company

Business income of a resident person shall be Pakistan source income if it is derived from any business
carried on in Pakistan. [Section 101 (2)]

Where any expenditure relates to the derivation of income chargeable to tax under the head income
from business (Normal) and Other business (Speculation), the expenditure shall be apportioned on any
reasonable basis taking account of the relative nature and size of activities to which the amount
relates [Section 19 (1) (c) + Section 67 + Rule 13 of Income Tax Rules, 2002]

Where a person has been allowed a tax deduction in respect of any expenditure and subsequently the
person receives the amount of such expenditure as a reimbursement in cash or in kind, the amount so
received shall be included in the person's income chargeable to tax in the year in which it is received.
(Section 70)

Carry forward of losses:
Loss for a tax year under this head (other than loss from a speculation business) shall be set off against
any other head of income but can be carried forward only against future business income up to 6 tax
years immediately succeeding the tax year in which the loss occurred. The loss of the earlier tax year
shall be set off first. (Section 57)

Speculation business loss can be set off against any gain arising from speculation business in the tax
year in which the loss arises and can be carried forward only against future speculation gains up to 6
tax years immediately succeeding the tax year in which the loss occurred. The loss of the earlier tax
year shall be set off first. (Section 58)
Part of the tax depreciation (including initial allowance, first year allowance or accelerated
depreciation) which has not been set off against income shall be carried forward in the normal manner
in the following tax year and so on until it has been completely set off. These deductions shall be taken
into account last. (Section 57)

Notes on income tax laws
Tax Year 2012
Muhammad Ovais, Deloitte 13th MFC


15

Deductions permissible in arriving at the income chargeable to tax under the head Income from
business: (Section 20)
1. A deduction shall be allowed for expenditure incurred by the person exclusively for the purpose
of business.

2. Deduction equal to the amount of Cost of animals used for the purpose of business (otherwise
than as stock in trade) and the Amount realized in respect of carcasses or animals shall be
allowed where the animals have died or become permanently useless for such purposes

3. Amortization or depreciation of intangible or tangible assets where they have a useful life of
more than 1 year (Section 22, 23, 24), (Third Schedule)

4. Pre-commencement expenditure at the rate of 20% on straight line basis (Section 25), (Third
Schedule)

5. Scientific research expenditure (Section 26)

6. Employee training facilities expenditure (Section 27)

7. Profit on debt if related to Taxable Business Income (Section 28)

8. Entertainment expenditure in the limits as prescribed (Rule 10 of Income Tax Rules, 2002)

9. Bad Debts (Section 29)

Explanation for the above mentioned items is as under:

3. Depreciation & Amortization:
a. Depreciation: (Section 22, 23)
i. Depreciation shall be allowed in relation to depreciable assets used in relation
to the person's business

ii. Depreciation rates are specified below: (Third Schedule)
Description of Asset Rate of Depreciation
Building 10%
Furniture, plant and machinery, motor vehicles, ships,
technical or professional books
15%
Computer hardware and aircrafts and aero engines 30%
Ramp built to provide access to disabled persons not
exceeding 250,000 each
100%

iii. Depreciation shall be allowed on proportional basis if the asset was also used
for the purpose other than deriving business income in a tax year.

iv. WDV of the asset in the above case shall be computed on the basis that the
asset has been solely used to derive business income. It means that
depreciation allowed as well as disallowed shall be deducted from the cost of
the asset in arriving at the WDV. In that case, the WDV of the asset shall be
increased by the amount of depreciation disallowed on account of non business
use at the time of disposal.

v. Depreciation shall not be allowed in the year of disposal
Notes on income tax laws
Tax Year 2012
Muhammad Ovais, Deloitte 13th MFC


16


vi. For computing gain on disposal of immoveable property, the consideration
received shall be treated as the cost of the property if the consideration
exceeds its cost (Gain on disposal shall be equal to the depreciation allowed)

vii. For computing gain on disposal of a depreciable asset by way of export that has
been previously used in Pakistan, the consideration received shall be treated as
the cost of the asset (Gain on disposal shall be equal to the depreciation
allowed)

viii. Initial Allowance equal to 50% of the cost of the asset shall be allowed on
eligible assets as defined below:
a. Road transport vehicle plying for hire
b. Building
c. Plant and machinery not previously used in Pakistan
d. Computer hardware
e. Technical and professional books

ix. First Year allowance in lieu of initial allowance equal to 90% of the cost of
the asset shall be allowed in respect of plant & machinery installed by any
industrial undertaking set up in specified rural areas and owned and managed
by a company

x. Accelerated Depreciation in lieu of initial allowance equal to 90% of the
cost of the asset shall be allowed in respect of plant & machinery installed
for generation of alternate energy by an industrial undertaking set up
anywhere in Pakistan and owned and managed by a company.

b. Amortization: (Section 24)
i. Amortization shall be allowed for the cost of intangible assets that have been
used for the purpose of deriving business income and that have a normal useful
life of more than 1 year

ii. An intangible with a normal useful life of more than 10 years or having an
unascertainable useful life, it shall be treated as having a normal useful life of
10 years

iii. Where intangible asset have been used partly for deriving business income,
amortization deduction shall be allowed proportionately based on the number
of days the intangible is used in deriving business income

iv. No amortization shall be allowed in the year in which the person disposes off
an intangible.

7. Profit on Debt: (Section 28)
x If the debt is utilized for business purpose

x Lease rentals of an asset to a financial institution, approved modaraba or leasing
company or Special Purpose Vehicle (SPV) on behalf of an originator

Notes on income tax laws
Tax Year 2012
Muhammad Ovais, Deloitte 13th MFC


17

x Financial cost of securitization of receivables by an originator in respect of SPV

x Share of profit under musharika scheme to a bank

x Share of profit to a certificate holder under a musharika scheme approved by SECP
and Religious Board under Modaraba Ordinance

x On funds borrowed from a modaraba or participation term certificate holders

x By a bank to a person maintaining PLS account or a deposit with the bank

x S8P's share of profit by H8FC, National 0evelopment Leasing Corporation or Small
and Medium Enterprises Bank on any investment or credit line provided by the SBP.

8. Entertainment Expenditures: (Rule 10 of Income Tax Rules, 2002)
a. A deduction for entertainment expenditures shall be limited to the following
expenditures directly related to the person's business:
i. incurred outside Pakistan on entertainment in connection with business
transactions
ii. incurred in Pakistan for foreign customers and suppliers
iii. incurred for customers and clients at the business premises
iv. incurred on entertainment at meetings of shareholders, agents, directors or
employees
v. incurred at the opening of branches

9. Bad Debts : (Section 29)
a. Allowable subject to the following conditions:
i. The amount of debt was previously included in the person's income chargeable
to tax or in respect of money lent by a financial institution deriving business
income chargeable to tax
ii. The debt is written off in the accounts
iii. There are reasonable grounds to believe that the debt is irrecoverable

b. Where the person receives the amount in cash or in kind, in respect of the debt which
has been allowed as a deduction in prior years, the following rules shall apply:
i. Where the recovery is in excess of the bad debts disallowed, the excess shall
be taxable in the tax year in which it is received
ii. Where the recovery is less than the amount of bad debts disallowed, the
shortfall shall be allowed as deduction from income from business in the year
in which it is received.
iii. Partial Recovery of Bad Debts: (Illustration)
Accounting Taxation Taxation
2010 Bad debt expense Admissible
(No effect)
In admissible
(Add)
2011 Bad debt recovery
(Other income)
Taxable
(No effect)
Not taxable
(Less)


Notes on income tax laws
Tax Year 2012
Muhammad Ovais, Deloitte 13th MFC


18

Deductions inadmissible in arriving at the income chargeable to tax under the head Income from
business: (Section 21)

Particulars Accounting
treatment
Tax
treatment
Effect
Expenditure Admissible Admissible No effect
Expenditure Admissible In admissible Add
Expenditure In admissible Admissible Less

Income Taxable Taxable No effect
Income Taxable Exempt Less
Income Exempt Taxable Add

x Any cess, rate, or tax paid or payable in Pakistan or foreign country on the profits of the
business as a % or otherwise on the basis of such profits (Income tax charge, WWF, WPPF)

x Any amount of tax deducted at source

x Any payment of salary, rent, commission/brokerage, profit on debt, services and payments to
non-residents made without tax deduction where a person is required to deduct tax.

o Below is the table which shows necessary tax deduction requirements:
Tax deducting agency Salary Rent Brokerage/
Commission
Profit
on debt
Payment to
Non-
resident
Services
Section 149 155(3) 233 151 152 153(7)
Company

AOP Reg. firm





AOP URF turnover of 50m
or more



AOP URF turnover < 50m




Individual having turnover
50m or more



Individual having turnover
< 50m





x Entertainment expenditures in excess of limits provided in Rule 10 of Income Tax Rules, 2002

x Any fine or penalty for the violation of any law, rule or regulation

x Personal expenses of the tax payer

x Amount carried to reserve fund or capitalized in any way

x Any profit on debt, brokerage, commission, salary or other remuneration paid by an AOP to a
member of the association


Notes on income tax laws
Tax Year 2012
Muhammad Ovais, Deloitte 13th MFC


19

Illustration:
Accounting Profit 5,000,000
Add:
Accounting Depreciation 150,000
Tax gain on disposal 20,000
Salary to Partners 1,250,000
Commission to Partners 500,000
Less:
Tax Depreciation 200,000
Accounting gain on disposal 10,000
Taxable Income 6,710,000
Tax @ 25% 1,677,500
Divisible Income 5,032,500

Partner
Salary 1,250,000
Commission 500,000
Balance 3,282,500
5,032,500

x Payments of business expenditure required to be paid through banking channel other than the
following:
o Where salary of an employee does not exceed 15,000 per month
o Where aggregate of a single account head does not exceed 50,000 for the year
o Single payment up to 10,000
o Payment on account of freight, travel fare, postage, utility and other government dues

x Payments to establish a business entity (e.g. company incorporation expenses)

x Contribution to unrecognized provident fund, unapproved pension fund, unapproved
superannuation fund or unapproved gratuity fund unless the person has made effective
arrangements to secure that tax is deducted from any payments made by the fund.
o When gratuity is actually paid to an employee from any gratuity scheme or unapproved
fund, the amount paid constitutes an admissible deduction for the income year in
which it is paid (Circular 11 of 1980)

Business Assets: (Section 75-79)
Disposal of an asset also includes the disposal of part of an asset. A person shall be treated to have
disposed off the asset when the assets is sold, destroyed, exchanged, transferred, cancelled, lost,
expired etc. Application of business asset to personal use shall also be treated as disposal. (Section 75)

Cost of an asset purchased by a person shall include: (Section 76)
x Consideration paid including FMV of consideration given in kind

x Expenditure in acquiring and disposing off the asset

x Expenditure to alter or improve the asset

Notes on income tax laws
Tax Year 2012
Muhammad Ovais, Deloitte 13th MFC


20

x Currency exchange differences arising on a loan from which the asset is purchased shall also be
added to or deducted from the cost of the asset in the year of occurrence, including the
effect of any hedging agreement relating to the loan

x Amount chargeable to tax or amount exempt from tax charged in addition to amount paid at
the time of acquisition (amount charged to sales whether taxable or exempt from tax)

x As reduced by any grant subsidy, rebate, commission or any other assistance received or
receivable in respect of acquisition of the asset.

No gain or loss shall arise on disposal of the asset by reason of compulsory acquisition of the asset
under any law, where the consideration received for the disposal is reinvested by the recipient in an
asset of a like kind within one year of the disposal.
x In such case, the cost of replacement asset shall be cost of asset disposed off + (consideration
given for replacement asset in excess of consideration received for the asset disposed off)


Notes on income tax laws
Tax Year 2012
Muhammad Ovais, Deloitte 13th MFC


21

CAPITAL GAINS

1. Capital Asset has been defined as property of any kind connected with business or not, but
does not include:
a. Stocks, consumables or raw materials held for business
b. Depreciable asset or amortizable asset
c. Immoveable property
d. Moveable property held for personal use of the person or any dependent family
member [excluding capital assets mentioned u/s 38(5)]

2. Specified Capital Assets (Securities u/s 37A) includes:
a. Shares of a public company
b. Vouchers of PTCL
c. Modaraba Certificate
d. Redeemable capital
e. Derivative product

3. Public Company means:
x Company listed in Pakistan at the year end
x A company in which 50% or more shares are held by:
FG or PG; or
Foreign government; or
Foreign company wholly owned by foreign government

4. Other Capital Assets (Section 37) which may include shares of a private company, membership
card of a stock exchange, share in partnership firm or other personal assets other than as stock
in trade, depreciable or amortizable assets, immoveable property, moveable property held for
personal use of a person excluding other than those mentioned in section 38 (5). [Those assets
include painting, sculpture, drawing, jewelry, rare manuscript, postage stamps, first day
cover, coin, or an antique on which loss shall not be recognized on their disposal]

5. Where the capital asset is held for more than 1 year, other than those mentioned in section
37A, gain if any on their disposal shall be restricted to 75% (25% is exempt)

6. Where a person sustains loss on disposal of any of the capital asset mentioned u/s 37A, it shall
be set off only against gain arising from any other security mentioned u/s 37A, and any
unadjusted loss shall not be carried forward.

7. Capital gain on disposal of the shares of a company in Export Processing Zone (EPZ) is exempt
(Clause 114 2
nd
Schedule Part I)







Notes on income tax laws
Tax Year 2012
Muhammad Ovais, Deloitte 13th MFC


22

8. Taxability of capital gain on disposal of securities mentioned u/s 37A is as under: (1
st

Schedule)
Holding
Period
Tax Year Rate of
Tax
Less than 6
months
2011 10%
2012 10%
2013 12.5%
2014 15%
2015 17.5%

9. Where the capital asset is transferred by
way of:
a. Gift, bequest or a will
b. Succession, inheritance or devolution
c. Distribution of assets on dissolution of an AOP
d. Distribution of assets on liquidation of a company

No gain or loss shall arise where the recipient is a resident in Pakistan in the relevant tax year. The
recipient shall be treated to have acquired the capital asset at the FMV at the time of such transfer.

10. Capital gain on bonus shares subsequently disposed off representing the difference between
consideration received and the dividend in specie (face value of bonus shares) is taxable in
accordance with section 37 or 37A. (Clause 103B, Part I, 2
nd
Schedule)

Illustration:
Consideration received for all shares xxxx
Less:
Cost of shares other than bonus shares xxxx
Face value of bonus shares xxxx
Capital gain/loss xxxx

11. Any gain from the alienation of any share in a company, the assets of which comprise wholly or
principally, directly or indirectly, of immoveable property or rights to explore natural resources
in Pakistan shall be Pakistan source income. (Section 101)
a. It means than if a non-resident company is involved in exploration of natural resources
in Pakistan wholly or substantially or where a non-resident company derives income
from lease of immoveable property in Pakistan, then capital gain on shares of such a
company is a Pakistan source income and a non-resident shareholder is taxable in
Pakistan subject to tax treaty.

11. Any gain arising on the disposal of shares in a resident company shall be Pakistan source
income.

12. Consideration in calculation of capital gain:
x Capital loss, if any, shall not be restricted to 75% and therefore, the full amount of loss shall be
adjusted or carried forward.
x Shares held will be considered as capital asset even if they are held as stock in trade, and the
gain on disposal will be taxed as capital gain (Circular 2-IT/1972 dated 01/07/1972)
Holding
Period
Tax Year Rate of
Tax
More than 6 months
but less
than 12 months

2011 7.5%
2012 8%
2013 8.5%
2014 9%
2015 9.5%
2016 10%
More than 12 months - 0%
Notes on income tax laws
Tax Year 2012
Muhammad Ovais, Deloitte 13th MFC


23

Carry forward of losses:
Net Capital loss can be carried forward against future capital gains up to 6 tax years immediately
succeeding the tax year in which the loss occurred. The loss of the earlier tax year shall be set off first.
(Section 59)


Notes on income tax laws
Tax Year 2012
Muhammad Ovais, Deloitte 13th MFC


24

INCOME FROM OTHER SOURCES


a. Income of every kind received by a person (taxable on receipt basis) in a tax year, if it is not
included in any other head, other than income exempt from tax under the ordinance, shall be
chargeable to tax in that year under the head Income from Other Sources including the following:

1. Dividend (including face value of bonus shares i.e. dividend in specie at the time of
disposal of bonus shares)

2. Royalty (foreign source royalty received by a company registered under Companies
Ordinance, 1984 and having its registered office in Pakistan is exempt: Clause 131 Part I 2
nd

Schedule)

3. Profit on debt

4. Ground rent

5. Rent in respect of lease of building together with plant and machinery

6. Amount received for the provision of amenities, utilities and any other service connected
with renting of the building

7. Prize bond, winnings from a raffle, lottery, prize on winning a quiz, prize offered by
companies for promotion of sale or cross word puzzle (Prize bond & Cross-word puzzle is
taxable @ 10% of gross amount; Raffle, lottery, prize on quiz and offered by companies
for promotion is taxable @ 20% of gross amount) [1
st
Schedule]

8. Amount received as a loan, advance, deposit for issuance of shares, gift by a person
otherwise than by banking channel other than advance payment for sale of goods or supply
of services.
x Cash Loan: (Circular 12, 1992)
It is only the peak credit of the lender which is to be taken as deemed income of
the tax payer and not the aggregate of all sums of loan received during the
relevant year.

9. Where a person fails to provide a reasonable explanation:

(1) For any amount credited in a person's books,
(2) For source of funds where a person has made an investment or is the owner of any
money or valuable article,
(3) For any expenditure incurred or
(4) Where a person has concealed income or furnished inaccurate particulars of income
including:
o The suppression(concealment) of any production, sales or any amount chargeable
to tax; or
o The suppression(concealment) of any item of receipt liable to tax in whole or in
part

Notes on income tax laws
Tax Year 2012
Muhammad Ovais, Deloitte 13th MFC


25

The unexplained amount shall be included in the person's income chargeable to tax under
the head income from other source to the extent it is not adequately explained. (Section
111)
a. Foreign exchange remitted from abroad through normal banking channels and got
en-cashed in Pakistan rupee from a scheduled bank is immune and no question shall
be asked
b. Section 111 shall not apply on encashment of Foreign Exchange Bearer Certificates
(FEBC), US $ Certificates and Foreign currency bearer certificates.

10. Where the declared cost of an asset is less than the reasonable cost of that asset, the
commissioner may having regard to all the circumstances include the difference in the
person's income chargeable to tax under the head income from other source. (Section
111)

b. Where any profit on debt derived from National Savings 0eposit Certificate including 0SCs' is paid
to a person in arrears and as a result his income is chargeable to higher rate of tax than would have
been applicable if the amount had been paid in the tax year to which it relates, he may by a notice
in writing to the commissioner by the due date for furnishing employees return of income, elect for
the amount to be taxed at the rates that would have been applicable if the amount had been paid
in the tax year to which it relates.

c. A deduction shall be allowed for any expenditure incurred in deriving income chargeable to tax
under the head income from other source, such as the following:
i. Zakat deducted in accordance with Zakat & Ushr Ordinance, 1980 from profit on debt
ii. Depreciation (including initial allowance) of plant and machinery against Rent in respect of
lease of building together with plant and machinery.


Notes on income tax laws
Tax Year 2012
Muhammad Ovais, Deloitte 13th MFC


26

LOSSES

Loss under any head of income can be set off against any other head of income specified in section 11,
other than speculation loss and capital loss

Loss under the head 'ncome from 8usiness' shall be set off last in priority of loss under another head
of income. (Section 56)

Loss incurred by an AOP shall not be available for its members and the AOP shall carry forward its
losses in the subsequent tax years in the normal manner (Section 59A)

Foreign Losses: (Section 104)
Foreign losses shall be carried forward only against foreign source income up to 6 years immediately
succeeding the tax year for which the loss was computed. This is based on the premise that foreign
source income chargeable under a head of income shall be treated as a separate head of income.

Business loss with reference to amalgamation: (Section 57A)

Assessed loss (excluding capital loss) for the tax year of the amalgamating company(s) shall be set off
against business profits or gains of the amalgamated company in the year of amalgamation subject to
the condition that the amalgamated company carries on the business of amalgamating company for a
minimum period of 5 years from the date of amalgamation. Any unadjusted loss can be carried forward
up to a period of 6 years succeeding the year of amalgamation.

Unabsorbed depreciation can be carried forward in the normal manner with no time limits.

However in case of amalgamation of banking company, non banking finance company, modaraba or
insurance company, the accumulated business loss (other than speculation loss) can be adjusted &
carried forward in the manner mentioned above.

Non compliance with any of the conditions laid down by SBP, SECP or any court shall render the
allowed adjusted loss be treated as the income of the amalgamated company in the year which the non
compliance occurred.

Group Taxation: (Section 59AA)
Holding companies and 100% owned subsidiaries locally incorporated under the Companies Ordinance,
1984 may opt to be taxed as a single fiscal unit for which consolidated group accounts shall be required
for the computation of income and tax liability. Such option exercised by the group companies shall be
irrevocable. Such relief shall not be available to losses prior to the formation of group.

Inter-corporate dividend income within the group companies shall be exempt.







Notes on income tax laws
Tax Year 2012
Muhammad Ovais, Deloitte 13th MFC


27

Group relief: (Section 59B)

A subsidiary company may surrender its tax loss (excluding b/f loss and capital loss) in favor of its
holding company or in favor of any subsidiary of its holding company. The holding company shall
directly hold share capital of the subsidiary company as under:
One of the company in group is public listed 55% or more
None of the company in group is public listed 75% or more

The loss surrendered by the subsidiary company may be claimed by the holding or any subsidiary
company in the tax year in which the loss has been surrendered and in the following 2 tax years subject
to the following conditions:

1. There is a continued ownership for 5 years of the share capital of the subsidiary company as
mentioned above (reversal of availed relief shall take place if the equity interest falls below
the minimum required in such 5 years)

2. A trading company within the group shall not be entitled to avail group relief

3. If a holding company is a private company, it shall get itself listed within 3 years from the year
in which the loss is claimed

4. Approval of BOD of both the companies (loss surrendering & loss claiming) is necessary

5. Subsidiary company shall continue the same business during the specified period of 3 years

6. All the companies in the group comply with corporate governance requirements.

The subsidiary company cannot surrender its assessed losses for more than 3 tax years. Any unadjusted
loss of subsidiary company after the specified period shall be carried forward by the subsidiary in the
normal manner. (3 consecutive tax years according to Circular 1 of 2007)

Inter-corporate dividend income within the group companies shall be exempt.

Loss claiming company may with the approval of BOD, transfer cash equal to the savings of tax in this
respect. Such amount shall not be allowable tax expense nor be taxable income for both the
companies.

Transfer of shares between companies and shareholders in one direction, would not be taxable capital
gain provided the transfer is to acquire share capital for the formation of a group and approval of SECP
or SBP has been obtained in this respect.


Notes on income tax laws
Tax Year 2012
Muhammad Ovais, Deloitte 13th MFC


28

TAX CREDITS

Tax credits shall be applied in the following manner:
x Foreign tax credit u/s 103
x Tax credit/rebate on donations, investment, enlistment, etc,
x Advance tax & tax deducted/collected at source

1. Senior Citizen Allowance to a person who is of 60 years of age or more on the 1
st
day of a tax
year and his taxable income does not exceed Rs. 1 million, shall be allowed a reduction equal
to 50% of his tax liability (Clause 1A, 2
nd
Schedule, Part III)

2. A full time teacher or researcher of a recognized nonprofit educational or research institution
including government training and research institutions shall be allowed a reduction of 75% of
tax liability on taxable salary only (Clause 2, 2
nd
Schedule, Part III)

3. Where a resident person derives foreign source income which is taxable in Pakistan, the tax
payer shall be allowed a tax credit in respect of foreign income tax paid by him as lower of
the following: (Section 103)
a. Foreign income tax paid
b. Pakistan tax payable in respect of foreign source income at the average rate of tax

Foreign income tax is to be paid within 2 years after the end of tax year to which it relates. If
not paid within 2 years, tax credit allowed earlier shall be treated as tax payable by the
person.

4. Charitable Donations in the form of any sum paid or property given by the person as a
donation to the following: (Section 61)
a. Board of education or university established under federal or provincial law
b. Educational institution, hospital or relief fund established or run by the federal,
provincial or local government
c. Approved Non-profit organization

Rebate on donations made in cash shall only be allowed if paid by a crossed cheque drawn on a
bank.

Tax credit shall be allowed at the average rate of tax on lower of the following:
x Actual amount of donation or FMV of the property given
x 30% of taxable income of individual or AOP (20% in case of a company)

Amount paid as donation to the institutions mentioned in clause 61, 2
nd
schedule are straight
deduction from the total income of the donor (treated like deductible allowances). Provided that the
maximum limits of 30% or 20% shall apply in the normal manner. The condition of payment through
banking channel is not applicable for donations to be permissible deductions.







Notes on income tax laws
Tax Year 2012
Muhammad Ovais, Deloitte 13th MFC


29

5. An eligible person (an individual Pakistani holding valid NTN, CNIC or NICOP) deriving income
chargeable to tax under the head salary or income from business shall be entitled to a tax
credit on the average rate of tax in respect of contribution or premium paid to an approved
pension fund. Tax credit shall be allowed on lower of the following: (Section 63)
a. Actual amount of contribution or premium; or
b. 20% of taxable income of the relevant tax year. Provided that an eligible person joining
the pension fund at the age of 41 years or above shall be entitle to additional 2% for
every year of age exceeding 40 years subject to the maximum of 50% of the taxable
income of the preceding year.

6. A resident person other than a company shall be entitled to a tax credit at the average rate of
tax in respect of purchase of the following shares: (Section 62)
a. Shares of a public listed company as an original allot tee
b. Shares acquired from the privatization commission of Pakistan

Amount eligible for tax credit shall be lower of the following:
x Actual cost of the shares
x Rs. 500,000
x 15% of the taxable income for the year

Shares are required to be held for at least 36 months otherwise tax credit allowed earlier
shall be reversed.

7. A resident person other than a company shall be entitled to a tax credit at the average rate of
tax in respect of any life insurance premium paid on a policy to a life insurance company
registered by the SECP under the Insurance Ordinance, 2000, provided the resident person is
deriving income chargeable to tax under the head "salary" or "income from business". (Section
62)

Amount eligible for tax credit shall be lower of the following:
x Total contribution of premium paid by the person
x Rs. 500,000
x 15% of the taxable income for the year


8. A person shall be allowed a tax credit in respect of profit on debt (including share in
appreciation in value of house) on a loan from a scheduled bank or a non banking finance
institution regulated by SECP or by government or a statutory body or a listed company for the
acquisition of house or construction of a new house. Tax credit shall be allowed at the average
rate of tax on the lower of: (section 64)
a. Actual amount paid
b. 50% of taxable income
c. Rs. 750,000

9. A manufacturer registered under the Sale tax act, 1990 shall be allowed a tax credit equal
to 2.5% of his tax liability if 90% of his sales are to the persons registered under sales tax act,
1990. Such credit is in respect of income other than FTR and for this purpose he shall provide
complete details of the persons to whom the sales were made. (Section 65A)

Notes on income tax laws
Tax Year 2012
Muhammad Ovais, Deloitte 13th MFC


30

10. For Enlistment in any registered stock exchange in Pakistan in the year in which it is listed
(Tax credit equal to 15% of tax payable) [Section 65C]

11. If a company invests in purchase of plant & machinery for the purpose of BMR (Balancing,
Modernization & Replacement) of plant & machinery already installed, credit @ 10% of the
amount of such investment is allowed against the tax payable in the year in which such plant &
machinery is installed. This tax credit is allowed if the plant & machinery is purchased and
installed between 1.7.2010 and 30.6.2015. (Section 65B)
a. If the amount of tax credit is more than the tax payable, then excess shall be carried
forward for 2 subsequent tax years.

12. If a taxpayer being a company: (Section 65D)
a. Establishes a new industrial undertaking for manufacturing in Pakistan, or
b. Invests in purchase and installation of plant & machinery for the purpose of BMR
(Balancing, Modernization & Replacement) of plant & machinery already installed, in
an industrial undertaking set up in Pakistan and owned by it, with 100% equity owned
by it

Tax credit @ 100% of the amount of the tax payable shall be allowed to such company on or after
July 01, 2011, for a period of 5 years or commencement of commercial production, whichever is
later.

If the amount of tax credits is more than the tax liability, then no refund shall be allowed nor the
same is allowed to be carried forward or carried back (except in case of tax credit on BMR).

Example (Senior citizen and full time teacher allowance):
Mr. A aged 120 years
Tax year 2011
Taxable salary as full time teacher Rs. 220,000
Taxable capital gain Rs. 240,000

Solution:

Taxable salary 220,000
Taxable capital gain 240,000
Total Taxable income 460,000 (Non Salaried Case)

Tax liability @ 7.5% 34,500
Senior citizen allowance @ 50% (17,250)
17,250
Full time teacher allowance @ 75% (6,188)
Tax liability 11,062
Working:
Full time teacher allowance in respect of tax liability on taxable salary:
(220,000/460,000) * 17,250 = 6,188


Notes on income tax laws
Tax Year 2012
Muhammad Ovais, Deloitte 13th MFC


31

Example (Donations and Shares):
Mr. C aged 68 years
Taxable salary Rs. 300,000
Taxable capital gain Rs. 90,000

Solution:

Taxable salary 300,000
Taxable capital gain 90,000
Total income 390,000
Zakat (u/s 60) (5,000)
Total Taxable income 385,000 (Salaried Case)

Tax liability @ 1.5% 5,775

Tax liability under marginal relief
350,000 * 0.75% 2,625
35,000 * 20% 7,000
Tax liability 9,625
Tax liability whichever is lower: 5,775
Senior citizen allowance @ 50% (2,888)
2,887
Rebate on donation of Rs. 20,000 (150)
Rebate on investment in shares of Rs. 30,000 (337)
Tax liability 2,400

Working:
Rebate on donation = (A/B) * C
= (2,887/385,000) * 20,000 = 150
Rebate on investment in shares = (A/B) * C
= (2,887/385,000) * 45,000 = 337


Notes on income tax laws
Tax Year 2012
Muhammad Ovais, Deloitte 13th MFC


32

ASSOCIATION OF PERSONS

An AOP shall be liable to tax separately from its members, and where AOP has paid the tax, the
amount of share of profit received by a member out of income of AOP shall be exempt in the hands of
the members. But the same shall be included in the taxable income for rate purpose only. (Section 88,
92)

FBR has clarified that it is the divisible income (profit after tax) of AOP that will be included in the
taxable income of its members for rate purpose.

Share of loss from AOP is not adjustable against income of its members nor is it considered for rate
purpose.

The amount of tax payable by the individual shall be calculated as:
Tax liability on an individual's taxable income including share of profit from AOP x
Taxable income including share of profit from AOP


Tax credit to a company receiving share of profit from AOP:
Share of profit from AOP shall be included in the taxable income of a company and taxable in the
normal manner. However, the company shall be entitled to a tax credit calculated in the following
manner: (Section 88A)
Share of profit from AOP x
Taxable income of AOP

Actual taxable income
excluding share of
profit from AOP
Tax assessed on AOP
Notes on income tax laws
Tax Year 2012
Muhammad Ovais, Deloitte 13th MFC


33

DISPOSAL OF BUSINESS

By an Individual to a wholly owned Company: (Section 95)

Where a resident individual disposes off all the assets of a business to a resident company, no gain or
loss on disposal is to be accounted for when following conditions are satisfied:
x Consideration received for disposal is in the form of shares (other than redeemable shares) of
the company
x The transferor must beneficially own all of the issued share capital of the transferee company
immediately after disposal
x Company must undertake to discharge all the liabilities in respect of assets acquired by the
company
x Liabilities shall not exceed the transferor's cost of the asset at the time of disposal
x FMV of the shares received in the consideration must be substantially the same as the FMV of
the assets transferred less any liabilities in respect of assets disposed off
x Company must not be exempt from tax in the tax year in which the disposal takes place.

For the purpose of above conditions:
Cost of acquisition shall be:
x Tax WDV in case of depreciable/amortizable assets
x Lower of cost or NRV of stock in trade
x Transferor's cost, in any other case

7UDQVIHURUVFRVWLQUHVSHFWRIVKDUHVUHFHLYHGDVFRQVLGHUDWLRQ shall be:
x Cost of acquisition for a company
x Less: amount of liability that the company has undertaken to discharge in respect of that assets
x Divided by: number of shares received

Unabsorbed depreciation/amortization in respect of transferor's assets shall be allowed as a deduction
to the company in the tax year in which the transfer is made.

By an AOP to a wholly owned Company: (Section 96)

Where a resident AOP disposes off all the assets of a business to a resident company, no gain or loss
on disposal is to be accounted for when following conditions are satisfied:
x Consideration received for disposal is in the form of shares (other than redeemable shares) of
the company
x The AOP must beneficially own all of the issued share capital of the transferee company
immediately after disposal
x Vembers' of an A0P must have an interest in the shares of the company in the same proportion
as in the business assets immediately before disposal
x Company must undertake to discharge all the liabilities in respect of assets acquired by the
company
x Liabilities shall not exceed the A0P's cost of the asset at the time of disposal
x FMV of the shares received in the consideration must be substantially the same as the FMV of
the assets transferred less any liabilities in respect of assets disposed off
x Company must not be exempt from tax in the tax year in which the disposal takes place.
Notes on income tax laws
Tax Year 2012
Muhammad Ovais, Deloitte 13th MFC


34

For the purpose of above conditions:
Cost of acquisition shall be:
x Tax WDV in case of depreciable/amortizable assets
x Lower of cost or NRV of stock in trade
x Transferor's cost, in any other case

$23VFRVWLQUHVSHFWRIVKDUHVUHFHLYHGDVFRQVLGHUDWLRQ shall be:
x Cost of acquisition for a company
x Less: amount of liability that the company has undertaken to discharge in respect of that assets
x Divided by: number of shares received

Unabsorbed depreciation/amortization in respect of A0P's assets shall be allowed as a deduction to the
company in the tax year in which the transfer is made.

By a wholly owned Company to a wholly owned Company: (Section 97)

Where a resident company disposes off all the assets of a business to another resident company, no
gain or loss on disposal is to be accounted for when following conditions are satisfied:
x Both companies belong to a wholly owned group of resident companies at the time of disposal
x Transferee must undertake to discharge all the liabilities in respect of assets acquired by the
transferor
x Liabilities shall not exceed the transferor's cost of the asset at the time of disposal
x Transferee must not be exempt from tax in the tax year in which the disposal takes place.

For the purpose of above conditions:
Wholly owned group companies means:
x One of the company beneficially holds all the issued share capital of the other company, or
x A third party beneficially holds all the issued share capital in both companies.

Cost of acquisition shall be:
x Tax WDV in case of depreciable/amortizable assets
x Lower of cost or NRV of stock in trade
x Transferor's cost, in any other case

7UDQVIHURUVFRVWLQUHVSHFWRIVKDUHVUHFHLYHGDVFRQVLGHUDWLRQ shall be:
x Cost of acquisition for a transferee
x Less: amount of liability that the company has undertaken to discharge in respect of that assets

Unabsorbed depreciation/amortization in respect of transferor's assets shall be allowed as a deduction
to the company in the tax year in which the transfer is made.








Notes on income tax laws
Tax Year 2012
Muhammad Ovais, Deloitte 13th MFC


35

Under a Scheme of Arrangement or Reconstruction: (Section 97A)

No gain or loss shall arise on the disposal of asset from one company to another, by virtue of operation
of a Scheme of Arrangement and Reconstruction under the provisions of Companies Ordinance, 1984 if
the following conditions are satisfied:
x Transferee must undertake to discharge all the liabilities in respect of assets acquired by the
transferor
x Liabilities shall not exceed the transferor's cost of the asset at the time of disposal
x Transferee must not be exempt from tax in the tax year in which the disposal takes place
x The scheme is approved by HC, SBP or SECP as the case may be.

For the purpose of above conditions:
Cost of acquisition shall be:
x Tax WDV in case of depreciable/amortizable assets
x Lower of cost or NRV of stock in trade
x Transferor's cost, in any other case

Unabsorbed depreciation/amortization in respect of transferor's assets shall be allowed as a deduction
to the company in the tax year in which the transfer is made.


Notes on income tax laws
Tax Year 2012
Muhammad Ovais, Deloitte 13th MFC


36

ASSOCIATES
Definition: Section 85

All transactions between associates are required to be made at an arm's length basis. However, where
the transactions is not at arm's length basis, the commissioner may in respect of such transactions,
distribute, allocate or apportion income, deductions or tax credits so as to reflect the income that
would be realized by the associates in an arm's length transaction. (Section 108)

Transfer Pricing: (Rule 20-27 on Income Tax Rules, 2002)
Transfer pricing refers to the setting, analysis, documentation, and adjustment of charges made
between related parties for goods, services, or use of property (including intangible property). Transfer
prices among components of an enterprise may be used to reflect allocation of resources among such
components, or for other purposes. OECD Transfer Pricing Guidelines state, "Transfer prices are
significant for both taxpayers and tax administrations because they determine in large part the income
and expenses, and therefore taxable profits, of associated enterprises in different tax jurisdictions.

These rules are made in account of powers of the commissioner u/s 108. When exercising powers under
that section, the Commissioner may give regard to the following methods for determining the arm's
length result between the associated parties, based on the arm's length standard. The selection is
based on the judgment of the Commissioner as to the suitableness of the method:
x Comparable uncontrolled price method
x Resale price method
x Cost plus method
x Profit split method (used only where none of the above methods is applicable)

A controlled transaction is said to have meet the arm's length standard if the results of the arm's
length standard is such that would have resulted if the associated parties have entered in the same
transaction under the same conditions.

Comparable Uncontrolled Price Method:
The price charged or paid in a controlled transaction must be the same as those in a comparable
uncontrolled transaction.

Resale Price Method:
Whether the price charged to an associated party realizes the same gross margin as would have been
realized in the same transaction under the same terms with the unrelated party or transaction between
uncontrolled persons.

Following steps shall apply in determining arm's length result:
1. Determine the resale price of the goods acquired from the associate
2. 0educt resale gross margin from the resale price (amount that covers the person's selling and
other operating expenses)
3. Deduct other costs associated with the purchase of a product (such as custom duty)

The amount remaining as the result of the above is the arm's length result and this must also be the
transfer price of the goods between associates.

Notes on income tax laws
Tax Year 2012
Muhammad Ovais, Deloitte 13th MFC


37

Cost plus Method:
Following steps shall apply in determining arm's length result:
1. Determine the total cost incurred in controlled transaction
2. To this, add an appropriate cost plus markup to reflect an appropriate profit under such
market conditions

The amount remaining as the result of the above is the arm's length result and this must also be the
transfer price of the goods between associates.

Profit Split Method:
This method may be adopted when the transactions are interrelated so that the arm's length result
cannot be determined on a separate basis.

Profit from such transaction is divided amongst the associates in the same manner as if amongst
independent person entering into such transaction on an arm's length basis.

Profit can be split amongst associates using any of the following methods:
a) Contribution analysis: based on the functions performed by each associate
b) Residual Analysis: total profit from controlled transactions are divided as follows:
a. Each associates is allocated a basic return appropriate for the type of transaction
b. Residual profit is allocated on a reasonable basis as would in an independent
arrangement

Thin Capitalization: (Section 106)
x Any profit on foreign debt incurred by a FCRC or a branch of a foreign company operating in
Pakistan in excess of 3:1 foreign debts to foreign equity ratio at any time during the year shall
not be allowed as tax expense.

x This section shall apply in the following cases:
o In case of a FCRC other than a banking company or a financial institution; or
o Where interest income of a non-resident is exempt in Pakistan or taxable at a rate
lower than normal corporate tax rate.

x Foreign Controlled Resident Company (FCRC) means a resident company in which 50% or
more of the underlying ownership is held by a non-resident person either alone or together
with any associates

x Foreign Debt in relation to FCRC means the highest amount at any time in a tax year of the
sum of the following:
o Foreign Debt o/s to Foreign Controller
o Foreign Debt o/s to any non-associate where that non-associate has a balance o/s of a
similar amount of debt owed to FC.

x Foreign Equity means the aggregate at the beginning of the tax year of the following:
o Paid-up value of shares held by FC
o Proportionate share of acc. Profits, share premium, and revaluation surplus as it would
be entitled to FC in the event of the company being wound up
o As reduced by any debt obligation owed by FC to FCRC and proportionate share of
accumulated losses if any.
Notes on income tax laws
Tax Year 2012
Muhammad Ovais, Deloitte 13th MFC


38

PROCEDURE
Refer sections (114-146B) for administrative procedures relating to filing of returns, assessments,
appeals and collection & recovery of tax.

Wealth Statement: (Section 116)

Wealth statement along with the wealth reconciliation statement shall be filed by every resident
person being an individual with the return of income where taxable income for the year or last
assessed income is Rs. 1,000,000 or more. Wealth statement shall show the following:
x Total assets and liabilities of the person, spouse, minor children and other dependents
x Any asset transferred to any other person during the tax year and the consideration for the
transfer
x Total expenditure incurred by the person, spouse, minor children and other dependents and
details of such expenditure.

Wealth statement along with the wealth reconciliation statement shall also be filed by a person filing
statement under FTR and has paid tax amounting to 35,000 or more.

Provided that every member of an AOP whose share from the income of such AOP, before tax, for the
year is Rs. 1,000,000 or more, shall also furnish wealth statement and wealth reconciliation statement
for the year along with the return of income of the Association.

A person may file a revised statement before the amendment of assessment if he finds any mistake or
omission.


Notes on income tax laws
Tax Year 2012
Muhammad Ovais, Deloitte 13th MFC


39

MINIMUM TAX

Where in a tax year, the tax payable is less than 1% of the turnover or no tax is otherwise payable for
any reason whatsoever including the following: (Section 113)
x Loss for the year
x Exemption from tax
x Application of credits/rebates
x Claiming of allowances or deductions
x Set off of a loss of an earlier year; or
x Adjustment of minimum turnover tax paid in earlier years

By the following persons:
x Resident company
x Individual having turnover of 50 million or above
x AOP having turnover of 50 million or above

Their turnover for the tax year shall be treated as the income of the person chargeable to tax and such
persons shall pay tax @ 1% of their turnover instead of actual tax liability.

Provided that, the amount paid in excess of actual tax liability shall be carried forward for adjustment
against tax liability for 5 tax years immediately succeeding the tax year for which the amount was
paid.

Not applicable:
x Where a company has declared gross loss before set off of depreciation and other inadmissible
expenses
x In case of Modaraba and Non-profit organization
x Special purpose, nonprofit company engaged in securitization of receivables of PG

Turnover means:
x Gross sales or gross receipts, exclusive of:
o Sales tax
o Excise duty
o Trade discount shown on invoices
o Sales/receipts taxable under FTR
x Gross fee for services, commission and gross receipts from contracts excluding covered under
FTR
x Share of profit from AOP received by a company out of the above income excluding those
covered under FTR







Notes on income tax laws
Tax Year 2012
Muhammad Ovais, Deloitte 13th MFC


40

Tax collected at import stage: [Section 148(8)]
Tax collected @ 5% on import of edible oil and packing material for a tax year shall be minimum tax.

Electricity consumption: (Section 235)
Advance tax is payable on commercial and industrial electric bills as per slabs defined in Part IV of 1
st

schedule.

Tax collected up to bill amount of 30,000 per month shall be treated as minimum tax for person other
than a company and no refund or adjustment shall be allowed to this extent. However, tax collected
with electric bills shall be adjustable or refundable in the following cases:
x For a person other than a company, tax collected on bills exceeding 30,000 per month
x In case of a company, without any threshold.

Services: [Section 153(3)(b)]
Tax deduction at source @ 6% from gross amount of service income shall be considered as minimum
tax. It means than no refund or adjustment shall be allowed from the said deduction.

The provision of minimum tax @ 6% shall not apply for a company listed on a registered stock exchange
in Pakistan receiving income from services.

Notes on income tax laws
Tax Year 2012
Muhammad Ovais, Deloitte 13th MFC


41

FINAL TAX

Retailers having turnover up to Rs. 5 million: (Section 113A)
May opt to pay tax @ 0.5% of their turnover as final tax

All persons selling goods to general public for consumption including manufacturers, dealers, and such
like persons having turnover up to Rs. 5 million are entitled to avail benefit u/s 113A. (Circular 19 of
2004)

Retailers having turnover exceeding Rs. 5 million: (Section 113B)
Shall pay final tax on his turnover as follows:
Annual Turnover Rate of tax
Exceeds 5 million but does not exceed 10 million 25,000 + 0.5% of the turnover exceeding 5 million
Exceeds 10 million 50,000 + 0.75% of the turnover exceeding 10
million

Turnover on which tax has been deducted at source @ 3.5% at the time of supply shall not be
considered for the purpose of turnover under this section.






Advance Tax on Imports: (Section 148)
Collector of customs shall collect income tax from every importer @ 5% on value as increased by
custom duty, sales tax, and federal excise duty. However, the FBR has prescribed reduced rates for
some of the items mentioned below:
x 1% on import of fiber, yarns, fabrics and goods covered by the Zero Rating Regime of sales tax
notified by the Board
x 3% on the import of raw materials imported by an industrial undertaking for its own use
x 1% on import of gold, silver and mobile telephone sets.


Except in the following cases, tax collected at import stage shall become full and final tax on the
income of the importer arising from the imports.
x Import of fertilizer by manufacturer of fertilizer
x Import of vehicles in Completely Built Unit (CBU) condition by manufacturer of vehicles
x Import by a large import house
x Industrial undertaking importing goods as raw material, plant & machinery and equipment for
its own use.

Circular 1, 2008: 2% additional sales tax charged to commercial importer at import stage is in
lieu of value addition at local supply stage. Hence it should be included in the value for the
purpose of section 148. (Sales tax general order 3/2007)

x Retailer means a person (being an individual or AOP) selling goods to general public for the
purpose of consumption.
x Retailers would not be entitled to claim any adjustment of withholding tax collected or
deducted under any head during the year.
Notes on income tax laws
Tax Year 2012
Muhammad Ovais, Deloitte 13th MFC


42

Circular 14, 1997: Tax at import stage shall not be collected on re-import of those goods which
were exported but rejected by foreign buyer or unsold consignment sent abroad for
display/sale were brought back.

Tax at import stage shall not be paid on goods temporarily imported into Pakistan for
subsequent exportation and goods imported by direct and indirect exporters if they are
covered under various notifications (Clause 56, Part IV, 2
nd
Schedule)

Commercial Importer:
(Circular 05 of 2002) the supplier may give a written declaration to the effect that conditions for
establishing commercial imports have been fulfilled and no tax @ 3.5% should be deducted while
making payment of supplies. The payer may require furnishing of import documents such as original bill
of entry, etc.

Toll Manufacturing Activities: The courts have held that an importer importing goods for toll
manufacturing cannot be considered as a commercial importer. Toll manufacturing contracts are as
good as self manufacturing / self consumption.

Tax on Dividends: (Section 5 + 150 + Clause 17 2
nd
Schedules, Part II)
Tax shall be deducted at source from the gross amount of the dividend paid by the company at the
following rates specified:

Shareholder of a power project privatized by WAPDA or
a company set up for power generation
7.5% of the gross amount of
dividend
Other corporate and non-corporate shareholder 10% of the gross amount of dividend

Such tax is full and final tax for a non-corporate shareholder, where as dividend income of a corporate
share holder is taxable @ 10% in the normal manner. It means that a corporate shareholder can deduct
direct expenses, if any, from dividend income and then tax @ 10% shall be calculated as a separate
block of income (Section 8)

Dividend includes any distribution by a company to its shareholder:
x Of all or part of its assets including money to the extent of accumulated profits
x Of debentures or deposit certificates to the extent of accumulated profits
x On liquidation to the extent of accumulated profits immediately before its liquidation
x On reduction of capital to the extent of accumulated profits
x Advance or loan by a private company to a shareholder where the company is substantially
involved in money lending business
x Remittance of after tax profit of a branch of a foreign company operating in Pakistan, other
than a branch of Petroleum Exploration and Production foreign company operating in Pakistan.






Notes on income tax laws
Tax Year 2012
Muhammad Ovais, Deloitte 13th MFC


43

Profit on Debt: (Section 151)
Tax shall be deducted @ 10% on the amount of profit on debt after deducting zakat paid to a resident
person on the following (other than interest income which is exempt under the Ordinance):
x Profit on certificates under NSS including DSC and Post Office Saving Account.
x Bank profit including profit and loss (PLS) sharing account
x Profit on securities including bonds, certificate, debentures etc. issued by a company or
financial institution
x Profit of government securities other than issued by National Savings Centre.

Tax shall not be deducted on profit on debt on loan through a loan agreement.

Payment for goods and services: (Section 153)
Following are the prescribed persons for the purpose of this section:
x Federal government
x Company
x AOP constituted under the law
x Nonprofit organization
x Foreign contractor or consultant
x Consortium or joint venture
x Exporter or an export house (for the purpose of s/s 2)
x AOP & Individual having turnover of 50 million or more

Every prescribed person shall deduct tax while making payment for goods and services at the gross
amount of the following:
Sale of goods 3.5% (1.5% in case of rice, cotton seed or edible oil)
Rendering of services 6% (2% in case of transportation services)
Execution of contracts 6%

Income from services rendered or construction contract outside Pakistan shall be taxable @ 1% of gross
amount if such receipts are brought into Pakistan through normal banking channel (Clause 3 & 3A, Part
II, 2
nd
Schedule)

Tax shall not be deducted if payment in a financial year does not exceed Rs. 25,000 in case of supply of
goods and Rs. 10,000 in case of services and execution of contracts.

Important exemptions from tax deduction are:
x Lease payments or purchase of an asset under lease and buy-back agreement
x Sale of goods by a large import house
x Payments to government including local government
x Payment to an indirect exporter in respect of inland back to back LC
x Sale made by the commercial importer who has paid tax u/s 148 at the import stage
x Purchase by a manufacturer cum exporter (tax shall be paid in respect of goods sold in
Pakistan if local sales are in excess of 5% of export sales)
x Companies operating Trading Houses
Notes on income tax laws
Tax Year 2012
Muhammad Ovais, Deloitte 13th MFC


44

x Payments made to traders of yarn specified in zero-rated regime of sale tax
x On refund of security deposit
x Payment for securitization of receivables by SPV to the Originator

Tax deducted under this section on the income of a resident person or permanent establishment of a
non-resident person shall be:
x Final tax on the sale of goods, except on:
o Payments received by the manufacturer of goods in respect of supply of those goods; or
o Payments received by a public company listed on registered stock exchange in Pakistan
x Minimum tax for the rendering or providing of services
x Final tax on the execution of contracts, except on:
o Payments received by a public company listed on registered stock exchange in Pakistan
on account of execution of contracts

Non tax deducting agencies comprise of Individuals and AOP having turnover of less than Rs.
50 million.

Status of Tax Deducted on Supply of Goods:
Supplier Supply to tax deducting
agency
FTR / Normal
Individuals/AOP-manufacturing Yes FTR
Individuals/AOP-manufacturing No Normal

Individuals/AOP-local trading Yes FTR
Individuals/AOP-local trading No Normal

Listed company-
manufacturing/local trading
Yes Normal
Listed company-
manufacturing/local trading
No Normal

Supplier Supply to tax deducting
agency
FTR / Normal
Unlisted company-
manufacturing
Yes Normal
Unlisted company-
manufacturing
No Normal

Unlisted company-local
trading
Yes FTR
Unlisted company-local
trading
No Normal

Payment to non-resident media persons: (Section 153A)
x Every person making payment for advertisement services to a non-resident media person
relaying from outside Pakistan shall deduct tax from the gross amount @ 10%.
x Insurance premium or re-insurance premium paid to a non-resident is subject to tax deduction
@ 5% of the gross amount which shall be final tax liability of the recipient [Section 152 (1AA)]
Notes on income tax laws
Tax Year 2012
Muhammad Ovais, Deloitte 13th MFC


45


Exports: (Section 154)
Every authorized dealer in foreign exchange or a banking company shall at the time of realization of
foreign exchange proceeds on account of export of goods by an exporter (direct & indirect export i.e.
inland back to back LC) deduct tax @ 1% from the export proceeds.

Every authorized dealer in foreign exchange shall at the time of realization of foreign exchange
proceeds on account of commission due to an indenting commission agent deduct tax @ 5% from the
proceeds.

Every export house making payment on account of stitching, dying, printing, embroidery, washing,
sizing and weaving shall deduct tax @ 0.5% of the gross amount payable which shall constitute a final
tax on such transaction. [Section 153 (2)]

Circular 3 of 2009:
Section 154(3C) along with this circular is relevant for the export to any country including Afghanistan
and received in cash without tax deduction by the authorized dealers. It provides that in respect of
goods exported without Form 'E', the collector of customs shall collect tax @ 1% at the time of clearing
of such goods for export

Other references:
x Supply of goods against international tenders are considered as export and therefore taxable
under FTR (Circular letter dated 04/04/1992)

x Local sales of goods (manufactured for export) as well as waste material not constituting more
than 20% of such production may be treated as export sales at the option of the tax payer
(Circular 20 of 1992)

x Duty drawbacks in respect of exports already covered under FTR shall not be considered as
additional receipts. The amount of such drawbacks etc. shall be deemed to have been covered
for tax purpose under FTR (Circular 14 of 1993)

x Income from export of computer software, IT services or IT enable services is exempt up to
30.6.2016 (Clause 133 2
nd
Schedule Part I)

x Advance payment received against future exports shall be deemed to be "Export Proceed
Pealized" and shall be subject to tax deduction u/s 154 under FTR. (Circular 14 of 1993)

x Tax shall be deducted at the time of discounting of export bills, if any, by the banks. (Circular
letter dated 09.07.1992)

x The provision of WHT and FTR will not apply in respect of exports made by those
manufacturers whose income is already exempt from tax.
In view of above explanation, we can form an opinion that if an agriculturist exports directly
then his exports would be exempt and should not fall within the ambit of FTR. (Circular 20 of
1992)



Notes on income tax laws
Tax Year 2012
Muhammad Ovais, Deloitte 13th MFC


46

Prizes and Winnings: (Section 156)
Person paying prize on a bond, winnings from a raffle, lottery, prize on winning a quiz, prize offered by
companies for promotion of sales or cross-word puzzle shall deduct tax from the gross amount paid.

Where such prize is not in cash, the person shall collect tax on the FMV of such prize.
Prize and winnings Rate of
Tax
Prize bond, cross-word puzzle 10%
Winnings from raffle, lottery, prize on winning a quiz, prize offered by companies for
promotion of sales
20%

Petroleum products: (Section 156A)
Every person selling petroleum products to a petrol pump operators shall deduct tax from the amount
of commission or discount allowed to the operator @ 10% of the gross amount.

The commissioner may issue a WHT (Withholding Tax Exemption) certificate @ 3.5% where a petrol
pump operator supplies goods to a tax deducting agency. (Circular 11 of 2004)

Brokerage and Commission: (Section 233)
Tax shall be deducted @ 10% of the commission or brokerage paid by government including local
government, company or AOP constituted under the law which shall be considered as full and final tax
liability.

Tax rate shall be 5% in case of advertising agents (Clause 26, 2
nd
Schedule Part II)


CNG Stations: (Section 234A)
Full and final tax @ 4% is applicable on the amount of gas consumption charges. CNG stations shall not
be entitled to claim any withholding tax.

Royalty and Fee for Technical Services earned by a non-resident: (Section 6)
Pakistan source Royalty income and Fee for technical services earned by a non-resident is subject to
withholding tax @ 15% of the gross amount which is full and final tax liability.


Notes on income tax laws
Tax Year 2012
Muhammad Ovais, Deloitte 13th MFC


47

MISCELLANEOUS PROVISIONS

Tax Accounting: (Section 33-36)
x A company shall apply accrual basis of accounting for the purpose of determining income
chargeable to tax under the head income from business.

x The board may prescribe for any class of persons to account for income chargeable to tax
based on accrual or cash basis accounting.

x Person accounting for income chargeable to tax on cash basis accounting under the head
income from business shall record income when it is received and expense when it is paid.
Person accounting for income chargeable to tax on accrual basis accounting under the head
income from business shall record vice versa to cash basis.

x Person accounting for income chargeable to tax on accrual basis accounting, when allowed a
deduction of expense, shall pay off such liability within 3 years of the end of tax year in which
it was allowed as a deduction. If not paid within such specified time, tax deduction allowed
earlier shall be reversed in the first year following the end of 3 years.

x Subsequent payment of such liability as mentioned above shall be allowed as a deduction for in
the year of payment.

x The closing value for a person's stock in trade shall be lower of cost and NP7.

x Income on long term contracts accounted for on the basis of accrual accounting system shall be
recorded based on the percentage of completion method. Computation is as follows:



Exemptions and Tax Concessions: (Section 41-55)
x Agricultural income shall be exempt

x Salary received by an employee of foreign government as remuneration for services rendered
to such government shall be exempt provided that:
o Employee is the citizen of foreign country and not a Pakistani citizen
o Services are similar to those provided by employee of the FG in foreign countries; and
o Similar exemption is granted by the foreign country to the employees of FG providing
services in foreign country.

x Allowance/monetary reward provided by the president of Pakistan shall be exempt

x Profit on debt received by a non-resident on securities issued by a resident person shall be
exempt if:
o Persons are not associates
o Profit was paid outside Pakistan
o Security is widely issued outside Pakistan for the purpose of raising loan outside
Pakistan for use in a business carried on in Pakistan
o Security is approved by Board for this purpose.
Cost incurred before the end of a tax year
Total estimated cost at the commencement of the contract
Notes on income tax laws
Tax Year 2012
Muhammad Ovais, Deloitte 13th MFC


48


x Scholarships granted to meet educational expenses shall be exempt (otherwise than where the
scholarship is paid by the associate)

x Income received by spouse under agreement to live apart shall be exempt

x Income of FG, PG, or LG shall be exempt other than income from business derived by FG or PG
carried on outside its jurisdictional area.

x Income of a Special Purpose Vehicle

x Foreign source Royalty and fee for technical service derived by a company registered under
Companies Ordinance, 1984 AND Foreign source fee for technical service derived by any other
tax payer subject to the following conditions: (Clause 131, Part I, 2
nd
Schedule)
o Such income is received in Pakistan by or on behalf of the said company or other tax
payer; and
o Where such income is not brought into Pakistan in the year in which it is earned and
the tax is paid thereon, an amount equal to the tax paid shall be deducted from the
tax payable in the year in which such income is brought into Pakistan.

Losses: (Section 56A)
x Company registered in Pakistan or AJK, operating hotels in Pakistan or AJK, sustains a loss in
Pakistan or AJK under the head income from business, shall be entitled to set off the amount of
loss against the company's income in Pakistan or AJK.

General:
x Small Company:
A company registered under the Companies Ordinance on or after 01-07-2005 and which:
o Has paid-up capital plus undistributed reserves not exceeding Rs. 25 million;
o Has employees not exceeding 250 at any time during the year;
o Has annual turnover not exceeding Rs. 250 million; and
o Is not formed by the splitting up or the reconstitution of company already under
existence

x Special Purpose Vehicle (SPV)
o A public company having the prescribed amount of paid up capital, a trust or a body
corporate may be registered as SPV with SECP for the purpose of Securitization under
the Companies (Asset Backed Securitization) Rules.

o An originator including a leasing company and modaraba may transfer its receivables to
SPV in consideration of a mutually agreed payment by SPV who is entitled to collect the
receivables.

o Lease rentals of an asset, used by a lessee for his taxable income, to SPV on behalf of
an originator is allowable tax expense for the lessee (Section 28)

o Financial cost of securitization of receivables by an Originator in respect of SPV is an
allowable tax expense

Notes on income tax laws
Tax Year 2012
Muhammad Ovais, Deloitte 13th MFC


49

o Payment by SPV to an Originator in respect of securitization of receivables shall not be
subject to tax deduction [Section 153(5)(d)]

o Income of SPV is exempt (Clause 136, Part I, 2
nd
Schedule)

o If any income arises in the accounts of SPV after the completion of securitization
process, the same shall be returned to the Originator in the next income year and such
income shall be taxable in the hands of Originator

o Minimum tax is not payable by a special purpose, nonprofit company engaged in
securitizing the receivables of PG

o If a person deducts tax from payment to SPV on behalf of an Originator, the Originator
is entitled to get credit of such tax [Section 153(8)]

x A person shall be treated to have received the amount or benefit if it is: (Section 69)
o Actually received by the person
o Made available to the person; or
o Applied on behalf of a person, at the instruction of such person or under any law

x If any activity has ceased and subsequently any benefit is derived in cash or in kind from this
activity that is taxable, then it shall be taxable in the normal manner had the activity not
ceased and all the provisions of this ordinance shall apply accordingly. (Section 72)

x Any amount that is chargeable to tax is taxable on the basis of receivable, that amount shall
not be chargeable again on the basis that it is received and vice versa. (Section 73)

x An author may elect to treat the amount received as royalty in respect of the literary or
artistic work which continued for a period exceeding 24 months, as having been received in
that tax year and the preceding two tax years in equal proportion. (Section 89)

Income Splitting:
x Income from an asset transferred by way of revocable transfer is taxable in the hands of the
transferor. Revocable transfer means a transfer: (Section 90)
o It gives the power to the transferor for the re-transfer of the asset during the life of
the transferee; or
o It gives a right to the transferor to resume power over the asset

x Income from any asset transferred by a person shall be taxable in the hands of the transferor if
the asset is transferred to his spouse of minor child or to any other person for the benefit of his
spouse or minor child (other than a married daughter). However, the following shall not attract
the application of this provision where the asset is transferred by way of registration or
mutation and the asset is transferred: (Section 90)
o For an adequate consideration (direct or indirect funds not provided by the transferor);
or
o In connection with an agreement to live apart

Notes on income tax laws
Tax Year 2012
Muhammad Ovais, Deloitte 13th MFC


50

x Income of a minor child chargeable under the head Income from Business (other than business
acquired through inheritance) shall be chargeable to tax as the income of the parent with the
highest taxable income for the year. (Section 91)

Anti-Avoidance:
x To determine the tax liability the commissioner may: (Section 109)
o Recharacterise a transaction or an element of a transaction that was entered into as
part of a tax avoidance scheme
o Disregard a transaction that does not have substantial economic effect
o Recharacterise a transaction where the form of the transaction does not reflect the
substance.

x Where the owner of a security (bonds, certificates, debentures, stocks and shares) disposes off
the security and afterward reacquires the security and as the result of the transaction, any
income of the security is receivable by a person other than the owner, such income shall be
treated to be of the owner's and not of the other person. (Section 112)

Authorized Representative: (Section 223)
x An authorized representative may be:
o Principle Officer in case of an company or AOP
o A partner in case of a partnership firm
o Trustee
o Relative or current full time employee
o Bank officer
o Legal practitioner
o An accountant (ACA, ACMA, ACCA, member of ICAEW)
o Income tax practitioner having experience of 10 years or more in the capacity of
Income tax officer or a higher post
o Person responsible for accounting receipts and payments in case of government or
public international organization, etc.
o In case of a non-resident tax payer: an employee, person having business connection or
holds receipts belonging to the non-resident

x Following persons are not authorized to represent a tax payer:
o An insolvent during the period of insolvency
o An employee dismissed from the income tax department
o An employee resigned from the income tax department (for a period of 2 years after
resignation)
o An employee retired from the income tax department who was involved in the tax
payer's income tax proceedings within one year before his retirement (for a period of 1
year after his retirement)
o Person convicted of an offence or found guilty of misconduct in relation to income tax
proceedings
o A legal practitioner or an accountant found guilty of misconduct in a professional
capacity

Notes on income tax laws
Tax Year 2012
Muhammad Ovais, Deloitte 13th MFC


51

ADVANCE TAX AND DEDUCTION OF TAX AT SOURCE

Quarterly Advance Tax: (Section 147)
Tax payer with the latest assessed income of 500,000 or more is required to pay advance tax on
quarterly basis as under:

Company and AOP: [Section 147(4)]
(AxB/C)-D
A is the turnover for the quarter
B is the tax assessed for the latest tax year
C is the turnover for the latest tax year
D is the tax paid in the quarter

x A company or an AOP shall pay quarterly advance tax on 25
th
of September, December, March
and June.

x A company or an AOP shall also be liable to pay quarterly advance tax in the absence of latest
assessed income or declared turnover. They shall pay advance tax on the basis of quarterly
estimated turnover including the effect of minimum tax.

x A person other than an individual investor shall pay quarterly adjustable advance tax on capital
gains made from disposal of securities within 21 days of the end of the quarter as under:

Holding Period Rate of advance tax
Less than 6 months 2% of the capital gains derived during the quarter
More than 6 months but less than 12
months
1.5% of the capital gains derived during the
quarter

Individual: [Section 147(4B)]
(A/4)-B
A is the tax assessed for the latest tax year
B is the tax deduction or paid at source in the quarter

An individual shall pay quarterly advance tax on 15
th
of September, December, March and June.

The following shall not be considered for the purpose of this section:
x Income under FTR
x Income from property; and
x Salary Income










Notes on income tax laws
Tax Year 2012
Muhammad Ovais, Deloitte 13th MFC


52


If a tax payer files an estimate and his advance tax paid is less that 90% of tax liability for the
relevant tax year, he shall be liable to pay default surcharge [Section 205 (1B)].

Cash withdrawal from a bank: (Section 231A)
Every bank shall deduct tax @ 0.2% of the cash amount if the payment for cash withdrawal or the sum
total of the payments for cash withdrawal in a day exceeds Rs. 25,000.

Such tax shall not be collected in the case of withdrawal made by:
x Federal or provincial government
x Foreign diplomat or diplomatic mission in Pakistan
x Person who produces the certificate of exemption of income.

Private motor vehicles: (Section 231B)
Every motor vehicle registration authority of Excise and Taxation Department shall collect advance tax
at the time of registration of a new locally manufactured motor vehicle at following rates:
Engine Capacity Amount of Tax
Up to 850cc 7,500
851cc to 1000cc 10,500
1001cc to 1300cc 16,875
1301cc to 1600cc 16,875
1601cc to 1800cc 22,500
1801cc to 2000cc 16,875
Above 2000cc 50,000

This section shall not be applicable in the following cases:
x Federal, provincial or local government
x Foreign diplomat or diplomatic mission in Pakistan

Members of stock exchange: (Section 233A)
A stock exchange registered in Pakistan shall collect advance tax from its members on the following:
Transaction Type Tax Rate
Purchase of shares 0.01% of purchase value
Sale of shares 0.01% of sale value
Trading of shares 0.01% of traded value
Financing of carry over trades (Badla) 10% of the carry over charge
If the taxpayer who is required to pay advance income tax under this section, is of
the view that his income for the current tax year would be likely to be less than his
latest assessed income, he may file an estimate of income to the Commissioner and
can pay the advance tax for the current tax year accordingly.

If a company or an AOP is required to pay advance income tax under this section, is
RIWKHYLHZWKDWLWVLQFRPHIRUWhe current tax year would be likely to be more than
its latest assessed income, he may file an estimate of income to the Commissioner
and can pay the advance tax for the current tax year accordingly.
Notes on income tax laws
Tax Year 2012
Muhammad Ovais, Deloitte 13th MFC


53

Telephone users: (Section 236)
Advance tax shall be collected on the amount of telephone bill of a subscriber, prepaid cards for
telephones and sale of units through any electronic medium or whatever form (such as easy load) at
the following rates:
In the case of telephone subscriber (other than
mobile phone subscriber) where the amount of
monthly bill exceeds 1,000
10% of the amount exceeding 1,000
In the case of subscriber of mobile phone and
prepaid cards
10% of the amount of bill or sales price of prepaid
card or sale of unit through any electronic
medium or whatever form.

Purchase of air ticket: (Section 236B)
Advance tax shall be deducted @ 5% on the purchase of gross amount of domestic air ticket.

Tax collected under this section shall be adjustable. This advance tax shall not be collected in the case
of:
x FG or a PG;
x A person who produces a certificate from the Commissioner Inland Revenue that income of
such person during the tax year is exempt.


Notes on income tax laws
Tax Year 2012
Muhammad Ovais, Deloitte 13th MFC


54

INCOME OF A NON-RESIDENT PERSON

Permanent Establishment: For definition, see section 2(41)

A nonresident is liable to tax in Pakistan only in respect of its Pakistan source income. Section 101
clearly defines the situations where the income of a nonresident shall be treated to be Pakistan source
income. (Section 11)

Business income of a nonresident person shall be treated to be Pakistan source income to the extent to
which it is directly or indirectly attributable to: [Section 101 (3)]

x Permanent establishment of the non-resident person in Pakistan (Example: business profits
earned by the Permanent Establishment of a non-resident company)

x Sales in Pakistan of goods merchandise of the same or similar kind as those sold by the person
through a permanent establishment in Pakistan (Direct sale by non-resident company of the
goods similar to that dealt in by its Permanent Establishment)

x Other business activities carried on in Pakistan of the same or similar kind as those affected by
the non-resident through a permanent establishment in Pakistan; or

x Any business connection in Pakistan (Real and intimate relationship between business
activities carried on by a non-resident which yields profits or gains, and some activities
carried on in Pakistan which contributes directly or indirectly to the earning of such profits
or gains)

The taxability of business profits in Pakistan of a non-resident primarily depends upon the existence of
permanent establishment in Pakistan, whether such profits are directly or indirectly attributable to the
non-resident person.

The following principles shall apply in determining income of a permanent establishment in Pakistan of
a non-resident person chargeable to tax under the head Income from Business: (Section 105)

x Permanent Establishment of a non-resident person shall be treated as a distinct and separate
person, engaged in same or similar activities, under the same or similar conditions and dealing
wholly independently as against the principle that a non-resident and its Permanent
Establishment is one and the same person.
x All expenses including executive and administrative expenses whether incurred in Pakistan or
elsewhere for the purpose of business activities of the Permanent Establishment are allowed as
deductions in computing income chargeable to tax under the head Income from Business.

x There are disallowances of certain expenses paid or payable by the Permanent Establishment
to its head office or to another permanent establishment of the non-resident person (other
than that towards reimbursement of actual expenses incurred by the non-resident person to
third parties). Those expenses are:

o Royalty, fee or other similar payments for the use of any tangible & intangible assets
by the Permanent Establishment
o Compensation for any services including management service performed for the
Permanent Establishment
Notes on income tax laws
Tax Year 2012
Muhammad Ovais, Deloitte 13th MFC


55

o Profit on debt on moneys lent to the Permanent Establishment except in connection
with a banking business

x There are disallowances of certain items of income in the hands of Permanent Establishment
charged to the head office or to another permanent establishment of the non-resident person
(other than that towards reimbursement of actual expenses incurred by the Permanent
Establishment to third parties). Those items are:

o Royalty, fee or other similar payments for the use of any tangible & intangible assets
o Compensation for any services including management service performed by the
Permanent Establishment
o Profit on debt on moneys lent by the Permanent Establishment except in connection
with a banking business

x Allocation of head office expenditures to the Permanent Establishment incurred for and on
behalf of Permanent Establishment are allowable as to the lower of the following amounts:
o Actual expenses as allocated by the head office; and
o Allowable head office expense calculated in the manner prescribed below:




x Following expenses incurred by the head office / nonresident person for the purpose of
Permanent Establishment shall not be allowed as a deduction in computing income chargeable
to tax under the head Income from Business:
o Profit on debt incurred by the nonresident person on debt to finance the operations of
the Permanent Establishment
o Insurance premium paid or payable by the nonresident person in respect of such debt.

Service Income of a Non-Resident: [Section 101 (4)]

Where the business of a nonresident comprises of providing independent services (including
professional services and the services of entertainers and sports person), the Pakistan source income of
the person shall include any remuneration derived by the person in respect of such services where the
remuneration is:
x Paid by a resident person; or
x Borne by a permanent establishment of a non-resident person.

It is essential to note that the nonresident must be in the business of rendering independent services,
such as stock/share brokers and marketing agents operating outside Pakistan and working
independently for the residents/PE in Pakistan.

It needs to be borne in mind that the said sub section does not require the presence of a PE in
Pakistan.

Remuneration derived by professionals in their personal capacity, such as sports men, artists,
entertainers, doctors, lawyers, etc. is also considered as Pakistan source if such receipts are received
from a resident person or PE of a non resident person in Pakistan, with the effect that rendering of
such services whether in Pakistan or abroad is immaterial.

Turnover of Permanent Establishment in Pakistan x Total Head Office expenditure
Worldwide turnover
Notes on income tax laws
Tax Year 2012
Muhammad Ovais, Deloitte 13th MFC


56

Example: Mr. A, a resident of Pakistan, instructs ABC plc, a brokerage house in UK, to purchase shares
of a company listed in the UK Stock Exchange. Any commission paid by Mr. A to ABC plc would be
considered as its Pakistan source.

Royalty: [Section 101 (8)]

Definition: Section 2(54)

Royalty shall be Pakistan source if it is:
x Paid by a resident person (except where royalty is payable in respect of any right, property or
information used, or services utilized for the purpose of a business carried on by the resident
outside Pakistan through a Permanent establishment); or
x Borne by a Permanent Establishment in Pakistan of a nonresident person.

Example: A US based company, licenses its software to a Pakistani company. Payment of such licensing
fee by the Pakistani company would be regarded as Pakistan source royalty income for US based
company, since the payer of such royalty is a Pakistani resident.

Example: In the above example, the Pakistani company uses that software to operate its Dubai branch.
The payment of royalty is not regarded as Pakistan source, because the royalty is payable for the
purpose of a business carried on by the resident outside Pakistan.

Example: Suppose that Pakistani company is a branch of a UK based company. Royalty payment would
be regarded as Pakistan source for the recipient (US based company) because it is borne by the
Permanent Establishment of a non-resident person.

Taxability:
Pakistan source royalty shall be charged to tax @ of 15% of the gross amount of payment. Such tax shall
be final tax on the amount of royalty. (Section 6, 8, & Part I 1
st
Schedule). Following are the
exception where tax deducted shall not become full and final tax liability of the recipient: [Section
6(3) + Rule 18 of Income Tax Rules, 2002]

1. Where royalty is exempt from tax

2. Where payment of royalty is made in pursuance of an agreement made before 08.03.1980 in
which case royalty received by the nonresident person shall be treated as income from other
source against which deductions u/s 40 will be allowed

3. Where a payment is made
a. In pursuance of an agreement made on or after 08.03.1980
b. The non-resident has a PE in Pakistan
c. The property or right giving rise to royalty is effectively connected with the Permanent
Establishment of a non-resident in Pakistan.

n that case, royalty shall be treated as "ncome from 8usiness" of the Permanent
Establishment against which following expenses will be allowed:
x Expenses incurred in Pakistan to earn such income; and
Notes on income tax laws
Tax Year 2012
Muhammad Ovais, Deloitte 13th MFC


57

x Expenses incurred outside Pakistan in pursuance of such agreement, not exceeding 10%
of the gross amount of royalty.

4. In any other case, where the royalty is not subject to FTR scheme, the amount of royalty would
be subject to tax after allowing for the following expenses:
a. Expenses incurred in Pakistan to earn such income
b. Expenses incurred in Pakistan in respect of any work done in pursuance of such
agreement; and
c. Expenses incurred outside Pakistan in pursuance of such agreement, not exceeding 10%
of the gross amount of royalty.

Deduction of tax at source:
x Every person making payment on account of royalty shall deduct tax @ 15% from the gross
amount chargeable to tax as Pakistan source royalty [Section 152(1)]

x Where the royalty is not governed under FTR, the general withholding tax of 20% would apply.
[Part I, 1
st
Schedule]

x The nonresident may obtain a certificate from the concerned Commissioner to avoid deduction
of tax at such a higher rate, in such case the payer would deduct tax @ 6% u/s 153(1)(c)

x Where royalty income does not fall under FTR, the tax so deducted shall be adjustable against
the final tax liability of the tax payer
x Tax so withheld shall be deposited into the government treasury w/in 7 days from the end of
the week in which the payment is made (Rule 43 of Income Tax Rules, 2002)

Exempt royalty:
x The recipient (nonresident) shall file return of income u/s114 in order to claim such exemption
x He must also obtain an exemption certificate from taxation authorities, to avoid deduction of
tax at the time of receipt of payment.
x If tax is deducted by the payer, then such tax is required to be claimed as a refund.
x Pakistan source Royalty income may be exempt say, by virtue of Double Taxation Treaty that
exists between Pakistan and the country of residence of such nonresident person.

Fee for Technical Services:

Definition: Any consideration, whether periodical or lump sum, for the rendering of any managerial,
technical or consultancy services including the services of technical or other personnel, but does not
include:
x Consideration for services rendered in relation to a construction, assembly, or like project
undertaken by the recipient; or
x Consideration which would be the income of the recipient chargeable under the head salary

Example: A construction company has been hired to construct a building in Karachi. As part of the
agreement, the company is also required to supervise the construction and electrical work over the
building. Such service fee in respect of supervision of the work derived by the company would not be
considered as fee for technical service, but as normal income applicable to tax payer.

Notes on income tax laws
Tax Year 2012
Muhammad Ovais, Deloitte 13th MFC


58

Illustration: Where technical services are rendered as ancillary and subsidiary to allow the use of a
property or right giving rise to royalty income, then receipt in respect of such technical fee is treated
as UR\DOW\LQFRPH and not as fee for technical service.

Example: A company incorporated in Pakistan acquired software from Microsoft US for which it pays an
amount of Royalty for the right to use the software. Under the terms of the agreement, Microsoft US is
also required to provide trouble-shooting services to the Pakistani company in case there are any
problems with the software. Although, by default the providing of such ancillary service may be
regarded as technical and hence taxable as Fee for Technical Service, however since the same is
directly related to the use of the software against which royalty is being derived, the said service fee
would continue to be regarded as Royalty and not a Fee for Technical Service.

Illustration: Transfer of technology constitutes Royalty income of the service provider of such facility,
since it allows the use of technology to the user. However where there is no actual transfer of
technology, but the provider merely renders such technical services to the user, then payment in
respect of such service is termed as a Fee for Technical Service.

Fee for technical services shall be Pakistan source if it is:
x Paid by a resident person (except where the services are utilized for a business carried on
outside Pakistan through a Permanent Establishment); or
x Borne by a Permanent Establishment in Pakistan of a nonresident.

Taxability:
Pakistan source fee for technical service income shall be charged to tax @ of 15% of the gross amount
of payment. Such tax shall be final tax on the amount so paid. (Section 6, 8, & Part I 1
st
Schedule).
Following are the exception where tax deducted shall not become full and final tax liability of the
recipient: [Section 6(3) + Rule 18 of Income Tax Rules, 2002]

1. Where fee for technical services is exempt from tax

2. Where payment for services is made in pursuance of an agreement made before 08.03.1980, in
which case the amount received by the nonresident person shall be treated as Income from
Other Source against which deductions u/s 40 will be allowed

3. Where payment for services is made in pursuance of an agreement made on or after 08.03.1980
but before 04.05.1981 in which case amount received by the nonresident person shall be
treated as income from other source against which deductions u/s 40 will be allowed subject
the maximum of 20% of the amount of such fee

4. Where a payment is made
a. In pursuance of an agreement made on or after 04.05.1981
b. The non-resident has a PE in Pakistan
c. The technical services are rendered through Permanent Establishment of a non-
resident in Pakistan.

In that case, such income shall be treated as "ncome from 8usiness" of the Permanent
Establishment against which following expenses will be allowed:
Notes on income tax laws
Tax Year 2012
Muhammad Ovais, Deloitte 13th MFC


59

x Expenses incurred in Pakistan to earn such income; and
x Expenses incurred outside Pakistan in pursuance of such agreement, not exceeding 10%
of the gross amount of fee.

However, the non-resident has an option to have such income charged to tax under FTR by
filing a written declaration to the commissioner w/in 15 days of the commencement of the
contract, which declaration shall remain valid till the completion of the contract.

Deduction of tax at source:
x Every person making payment on account of fee for technical service shall deduct tax @ 15%
from the gross amount chargeable to tax as Pakistan source income [Section 152(1)]

x Where the fee for technical service is not governed under FTR, the general withholding tax of
20% would apply. [Part I, 1
st
Schedule]

x The nonresident may obtain a certificate from the concerned Commissioner to avoid deduction
of tax at such a higher rate, in such case the payer would deduct tax @ 6% u/s 153(1)(c)

x Where royalty income does not fall under FTR, the tax so deducted shall be adjustable against
the final tax liability of the tax payer

x Tax so withheld shall be deposited into the government treasury w/in 7 days from the end of
the week in which the payment is made (Rule 43 of Income Tax Rules, 2002)



Exempt fee for technical service:
x The recipient (nonresident) shall file return of income u/s114 in order to claim such exemption
x He must also obtain an exemption certificate from taxation authorities, to avoid deduction of
tax at the time of receipt of payment
x If tax is deducted by the payer, then such tax is required to be claimed as a refund
x Pakistan source Fee for Technical Service may be exempt say, by virtue of Double Taxation
Treaty that exists between Pakistan and the country of residence of such nonresident person.

Shipping and Air Transport Income: (Section 7 & 143)
Tax shall be imposed at the rates specified below on every non-resident person carrying on the business
of operating ships or aircraft as the owner or charter thereof in respect of gross amount received or
receivable:
x For the carriage of passengers, livestock, mail or goods embarked (on board) in Pakistan; and
x For the carriage of passengers, livestock, mail or goods embarked (on board) outside Pakistan

Shipping 8%
Air transport 3%

Tax imposed at the above mentioned rates shall be the full and final tax liability.

S-ar putea să vă placă și